Sie sind auf Seite 1von 107

Anesthesia and Preoperative Medicine


1) An elderly male with pain in his left hip is 3) A 24 week pregnant woman is in need of an
scheduled for a hip arthroplasty. He emergency appendectomy. All are
requests to have a spinal anesthetic. All are physiologic changes in pregnancy EXCEPT:
potential complications of a spinal a) Decreased gastroesophageal sphincter
anesthetic EXCEPT: tone
a) Hypotension b) Decreased haemoglobin
b) Infection at site c) Decreased coagulation factors
c) Hematoma at site d) Decreased Functional Residual Capacity
d) Local anesthetic toxicity e) Decreased Systemic Vascular
e) Nausea & Vomiting Resistance

2) All of the following are contraindications to


spinal anesthesia EXCEPT:
a) Raised intracranial pressure
b) Hypovolemia
c) Coagulopathy
d) Infection at site of needle insertion
e) Kyphosis
ANSWERS

1. E 2. E 3. C
Cardiology and CVS

1) Each of the following has been shown to 6) Which of the following is not a mechanism
improve mortality in heart failure patients for atrial fibrillation affecting cardiac function:
except: a) Decreased time for diastolic filling
a) Beta blocker b) Decreased ventricular compliance
b) ACEI c) Irregular ventricular response
c) Loop diuretic d) Loss of atrial contraction
d) Aldosterone antagonist e) Decreased time for coronary perfusion
e) Nitrate

7) Dressler's syndrome refers to:


2) Each of the following is a potential a) Migratory superficial thrombophlebitis
component of the management of both b) The clinical features of myocarditis
STEMI and UA/NSTEMI except: c) Delayed pericarditis post MI
a) PCI d) A preexcitation syndrome similar to
b) ASA WPW
c) Clopidogrel e) Cardiac myxoma
d) Beta blocker
e) Fibrinolysis
8) Each of the following is a cause of left axis
deviation except:
3) Which of the following is not an absolute a) Left anterior hemiblock
contraindication to exercise ECG stress b) Inferior MI
testing? c) WPW
a) Acute aortic dissection d) RV pacing
b) Acute MI within 2 days e) LBBB
c) Unstable angina not stabilized by
medical therapy
d) Asymptomatic aortic stenosis 9) Each of the following in and of itself is a
e) Symptomatic heart failure not controlled potential indication for AAA repair except:
by therapy a) Rate of enlargement 1.0 cm/yr
b) Comorbid HTN, COPD and smoking
c) Size = 5.5 cm in a male
4) According to the newest set of ACC/AHA d) Size = 5.5 cm in a female
Guidelines, which of the following is an e) Ruptured aneurysm
indication for infective endocarditis
prophylaxis before bowel surgery ?
a) Previous infective endocarditis
b) Mitral stenosis
c) Bicuspid aortic valve
d) Prosthetic cardiac valve
e) None of the above

5) Each of the following is an ECG feature of


acute MI except:
a) U wave
b) T wave inversion
c) ST depression
d) New LBBB
e) Tall "hyperacute" T waves
10) The subendothelium is the most vulnerable 14) Endocarditis in an I.V. drug user:
segment of the heart from an ischemic a) Is equally prevalent to that of the normal
standpoint. The major reason for this is: population
a) The highest oxygen utilization is in the b) Transthoracic Echocardiography is
subendocardium highly sensitive in diagnosing
b) Coronary flow to the subendocardium c) Is typically caused by S. pneumoniae
occurs almost completely during diastole d) Is typically found on the tricuspid valve
whereas other regions receive some flow e) Is typically found on the aortic valve,
during systole as well producing a systolic ejection murmur
c) The subendocardium has a diminished
aerobic capacity
d) There is less potential for 15) Nitroglycerin administered sublingually may
collateralization to the subendocardium contribute to the relief of myocardial
e) The ratio of capillary to myocyte is less in ischemic pain by each of the following
the subendocardium than in other mechanisms EXCEPT:
regions a) Coronary vasodilation
b) Decreased venous pooling resulting in
increased cardiac preload
11) Impaired coronary flow reserve is c) Reduced systemic vascular resistance
associated with each of the following d) Reduced ventricular volume
conditions EXCEPT:
a) Severe aortic stenosis
b) Severe systemic hypertension with left 16) A 48 year-old male is brought to the ED with
ventricular hypertrophy CPR being administered. The ECG shows
c) Severe mitral stenosis in the presence of electrical activity present but you cannot
atrial fibrillation palpate a carotid pulse. Which of the
d) A totally occluded coronary artery but following is not on the differential for
with excellent collateral supply from the pulseless electrical activity?
contralateral (i.e. opposite) coronary a) Alkalosis
artery b) Hypokalemia
e) An isolated 30% diameter stenosis of a c) Cardiac tamponade
coronary artery d) Hypothermia
e) Tension pneumothorax

12) Which of the following would NOT be part of


your plan for the treatment of acute 17) Which of the following is not a surgically
ventricular fibrillation? correctable cause of hypertension?
a) Electrical defibrillation a) Aortic coarctation
b) Lidocaine b) Renal artery atherosclerosis
c) Epinephrine c) Pheochromocytoma
d) Amiodarone d) Primary hyperaldosteronism (Conn
e) Manganese syndrome)
e) None of the above

13) Which of the following is not an aggravating


factor of congestive heart failure? 18) Which of the following findings is
a) Hypertension inconsistent with cardiac tamponade?
b) Thyrotoxicosis a) Hypotension
c) Alcohol b) Pulsus paradoxus
d) Inactivity c) Kussmaul sign
e) Arrhythmia d) Jugular venous distension
e) Muffled heart sounds
19) Which of the following is the most common 24) All of the following are causes of left axis
cardiac manifestation of systemic lupus deviation (LAD) on ECG except?
erythematosus? a) Inferior myocardial infarction
a) Myocarditis b) WPW / pre-excitation / bypass tract
b) Pericarditis c) Hyperkalemia
c) Myocardial infarction d) Mechanical shift: Expiration, high
d) Mitral valve prolapse diaphragm from pregnancy, ascites,
abdominal tumors, etc.
e) Aortic stenosis
20) Which of the following is false regarding the
clinical findings of significant aortic
stenosis? 25) All of the following are the causes of right
a) Cresendo-decresendo systolic murmur axis deviation (RAD) on ECG except?
with left ventricular hypertrophy a) Right ventricular hypertrophy
b) Diminished or absent S2 b) Right bundle branch block
c) Severe cases cause paradoxical c) Lateral infarction
splitting d) Hypercalcemia
d) Parvus-et-tardus carotid upstroke e) WPW / Preexcitation / Bypass tract
e) Radiates to axilla

26) All of the following are criteria for the


21) All of the following are signs of increasing CHADS2 score for assessing the risk for
severity of AS except: stroke among patients with atrial fibrillation
a) Late peaking of murmur except?
b) Diminished or absent S2 a) Heart Failure
c) Radiation to carotids b) High blood pressure
d) Increased apical carotid delay c) History of smoking
e) Lack of brachio-radial delay d) Over 75 years old
e) Diabetes mellitus

22) All of the following can lead to elevation of


Troponin except? 27) When wide complex tachycardia is seen on
a) Myocarditis an ECG, all of the following signs and
b) Hypothyroidism symptoms suggest ventricular tachycardia
c) Pulmonary embolism (VT) rather than supraventricular
d) Subarachnoid haemorrhage tachycardia (SVT)?
e) Extreme exercise a) Carotid massage and adenosine
terminate the arrhythmia
b) Cannon A waves present on physical
23) All of the following are causes for low exam
voltage on a 12-lead ECG except? c) History of CAD and previous MI
a) Hypokalemia d) Capture or fusion beats
b) Pericardial effusion e) AV dissociation
c) Thick chest wall, barrel chest, COPD,
orobesity
d) Generalized edema 28) All of the following are causes of prolonged
e) Hypothyroidism with myxedema QT interval except
a) Coronary artery disease
b) Hypercalcemia
c) Sotolol
d) Tricyclic antidepressants
e) Hypothermia
ANSWERS
7. C 13. D 19. B 25. D
1. C 8. E 14. D 20. E 26. C
2. E 9. B 15. B 21. E 27. A
3. D 10. B 16. A 22. B 28. B
4. E 11. E 17. E 23. A (hypocalcemia
5. A 12. E 18. C 24. E causes
6. B prolonged QT)
Clinical Pharmacology

1. Which one of the following statements is 2. Which of the following statement is FALSE
TRUE regarding the biotransformation of with respect to pharmacodynamics?
drugs: a) Efficacy is a measure of the ability of
a) Cigarette smoke and chronic alcohol a drug to elicit an effect at its receptor
consumption induce P450 enzymes b) Potency is a concentration of a drug
b) Phase I reactions increase the lipid needed to elicit a given effect
solubility of drugs c) Agonists are drugs that bind to
c) Phenytoin (Dilantin™) is a potent endogenous ligands and exert an
P450 enzyme inhibitor effect
d) Phase II reactions decrease the d) A partial agonist can be made into full
water solubility of drugs agonist by increasing its
e) Biotransformation is increased in the concentration
elderly e) Antagonists are drugs that have
affinity for its receptor, but no efficacy
ANSWERS

1. A 2. D
Dermatology

1) A 20-year-old female presented with a 5) 18 year-old female with initial onset of


single non-pruritic plaque on her back with pruritic rash characterized by excoriations,
inward pointing scale. She developed scaling and crusting and distributed on the
similar smaller papules and plaques on the extremities, neck and eyelids. Past medical
back over a one week period. The most history is significant for asthma and hay
appropriate management is: fever. The most likely diagnosis is:
a) Topical antifungal a) Scabies
b) Topical antibiotic b) Atopic dermatitis
c) Oral antifungal c) Contact dermatitis
d) Topical corticosteroids as needed d) Shingles
e) Oral antibiotic e) Dyshidrotic eczema

2) A 8-year-old black boy presented with a 6) 8 year-old black male comes in with an
round, scaly patch of hair loss on the asymptomatic erythematous eruption
occipital scalp with scale and occasional characterized by oval patches with collarette
pustules. The most appropriate scaling. It is distributed as a "Christmas
management is: tree" pattern on the back. Father states that
a) Topical antifungal there was originally one lesion on the
b) Topical antibiotic abdomen a few weeks prior. What is the
c) Oral antifungal most likely diagnosis:
d) Intralesional corticosteroid injections a) Pityriasis rosea
e) Oral antibiotic b) Tinea corporis
c) Lichen planus
d) Psoriasis
3) A 29-year-old woman presents with a 5 day e) None of the above
history of arthralgias and a purpuric eruption
on the lower legs. The most appropriate
initial management is: 7) 30 year-old female comes in with a soft
a) Topical cortyicosteroids smooth, glistening, erythematous nodule on
b) Oral corticosteroids her lower lip. She states that a few weeks
c) Compression of the lower legs prior she had some chapped lips with
d) Reassurance occasional bleeding. Now, the lips have
e) Urinalysis healed but this lesion arose suddenly in its
place. It is occasionally tender on pressure.
The most likely diagnosis is:
4) The following statements regarding contact a) HSV1
dermatitis are true EXCEPT: b) Cherry hemangioma
a) Phototoxic dermatitis following topical c) Pyogenic granuloma
application of creosote requires UV light d) Dermal nevus
b) Photoallergic contact dermatitis requires e) None of the above
UV light to be manifested
c) Contact eczema involves a type IV
delayed hypersensitivity reaction
d) Contact urticaria or hives is a common
form of dermatitis
e) Chemical burns by HCl and KOH may
result in an irritant contact dermatitis
8) An 11 year-old male comes in with 12) Which of the following is associated with
erythematous pustules, inflamed nodules thyroid disease?
and cysts with some scaring distributed on a) Neurofibromatosis
the face predominantly. Diagnosis of acne b) Vitiligo
vulgaris was given. Topical erythromycin c) Erythema nodosum
was used for 2 weeks, several months ago, d) Pemphigus vulgaris
with no response. What treatment would you e) Icthyosis vulgaris
prescribe now?
a) Isoretinoin immediately
b) Topical tretinoin
c) Topical benzoyl peroxide
d) Topical antibiotic other than erythromycin
e) Oral antibiotic
f) Oral antibiotic and topical tretinoin

9) A 40 year-old woman develops recurrent


papules and pustules in a symmetrical
pattern on her cheeks, nose, chin and
forehead. She blushes easily, especially
when consuming hot liquids, alcohol, or
spicy foods. The most likely diagnosis is:
a) Acne vulgaris
b) Perioral dermatitis
c) Acne rosacea
d) Seborrheic dermatitis
e) Carcinoid syndrome

10) Which of the following is not associated with


thyroid disease?
a) Dermatitis herpetiformis
b) Urticaria
c) Porphyria cutanea tarda
d) Vitiligo
e) Alopecia areata

11) All of the following are treatments for non-


scarring alopecia EXCEPT:
a) Spironolactone
b) Minoxidil
c) Hair transplantation
d) Intralesional triamcinalone
e) Finasteride
ANSWERS

1. D 4. D 7. C 10. C
2. C 5. B 8. F 11. D
3. E 6. A 9. C 12. B
Diagnostic Medical Imaging (Radiology)


1) CXR of a newborn showing a 5) CXR showing tension pneumothorax.
hypodensity in the left side. The child is Separation of visceral and parietal pleura
in respiratory distress. What is your initial (density), hyperlucent small dilated lung,
management? with lack of peripheral lung markings.
a) Needle aspiration of the left lung What is the management?
b) Intubation and ventilation 1 – needle aspiration
c) expectant therapy 2 – chest tube drainage
d) IV N/S bolus 3 – intubation
e) Thoracotomy 4 – air tight dressing
5 – surgery

2) CXR of a neonate few hours after birth. The a) 1 and 4


x-ray showing a full abdominal and chest b) 3 and 5
view. The abnormality in the chest is of a c) 1 and 2
large dense central shadow, which may d) All of the above
represent an infiltration in the lungs or a e) None of the above
large heart. The child is 32-wk gestation and
soon after birth he developed RDS. What is
your diagnosis? 6) CXR showing hyperlucent left lung
a) Hyaline membrane disease with mediastinal shift. The child is
b) Congenital heart disease coughing and wheezing. What is the
c) Pneumonia diagnosis?
d) Persistent fetal circulation a) TB infection
b) Pneumonia
c) Left hyperlucent lung syndrome
3) Knee x-ray of a 2-yr old child who has not (Swyer-James syndrome)
walked yet. It shows the tibia, which has d) Influenza
wide cupping at its proximal end and
obscure metaphysial plate. There is no
significant family history of any disease. 7) 5 yo child with stridor and respiratory
What is wrong with the child? distress. X-ray shows thumb sign. What is
a) Rickets the diagnosis?
b) Achondroplasia a) Acute epiglottitis
c) Hyperparathyroidism b) Acute asthma attack
d) Hypercalcemia c) Retropharyngeal abscess
d) Tracheo-laryngo-bronchitis

4) 4. 35yo pt with recurrent hemoptysis


and CXR with middle lobe infiltration. 8) Which of the following radiation has the
The most likely diagnosis is: deepest penetration in human tissue:
a) Bronchiectasis a) Alpha
b) Lung abscess b) Gamma
c) Chronic bronchitis c) X-ray
d) Emphysema d) Delta
e) UV
9) Patient fell on his hand, emergency x-ray 14) Which investigation is confirmatory for IBD?
was normal, but the patient complained of a) Barium enema
pain during wrist abduction. What is the b) Colonoscopy
proper management? c) CT
a) Explain to him that the pain will d) MRI
disappear after 2 weeks, its normal.
b) Bandage
c) Short cast plus x-ray in 15 days 15) A 70-year-old man is seen in your office for
d) NSAID's assessment one week after experiencing a
brief episode of left arm and left facial
weakness. Blood pressure in your office is
10) Patient after big operation develops sudden 140/80 mmHg. A CT scan showed no
dyspnea, chest pain, the most likely finding evidence of hemorrhage or space-occupying
on chest x-ray is: lesion. An EKG shows normal sinus rhythm.
a) Air under diaphragm Which one of the following is the most
b) Kerly B lines important investigation to order at this time?
c) Perihilar hyperdensity a) An magnetic resonance imaging (MRI) of
d) Normal the brain
e) Cardiomegaly b) An echocardiogram
c) Cerebral angiography
d) A chest x-ray
11) Patient with lower back pain, x-ray: anterior e) A carotid artery ultrasound
osteophytes at L4-L5 and subchondral
sclerosis, diagnosis:
a) Degenerative spondylitis 16) A 75 year old man fell down and injured his
b) Ankylosing spondylitis head. He is awake and not complaining of
c) Osteoarthritis neck pain. What investigation are you going
d) Rheumatoid arthritis to do first?
a) CT of the head
b) MRI of the head
12) A 23 year old patient has a single thyroid c) X-ray Skull
nodule. Which of the following will help d) C-spine x-ray
make a diagnosis of a malignancy?
a) Thyroid ultrasound
b) Cold nodule on thyroid scan 17) Which of the following are NOT radiological
c) Hot nodule on thyroid scan features of Rheumatoid Arthritis?
d) CT cervical spine a) Soft tissue swelling
b) Periarticular Osteopenia
c) Bony erosions
13) Child has stridor, fever, and a swollen, d) Osteophytes
painful neck. What investigation will you e) Joint subluxation
order?
a) CT scan of the neck
b) Lateral neck x-ray
c) C-spine x-ray
d) MRI
18) Which of the following can result in a ring- 19) A 75 year old man fell down and injured his
enhancing lesion on CT? head. He is unconscious. What investigation
1 – Brain metastases are you going to do first?
2 – Brain abscesses a) CT of the head
3 – High grade astrocytomas b) MRI of the head
4 – Demyelination c) X-ray Skull
d) C-spine x-ray
a) 1, 2 and 3 only
b) 1 and 2 only
c) 1, 2 and 4 only
d) 1 and 4 only
e) All of the above
ANSWERS

1. B 5. C 9. C 13. B 17. D
2. D 6. C 10. D 14. B 18. E
3. D 7. A 11. A 15. E 19. D
4. A 8. C 12. B 16. A
Emergency Medicine


1) A 27 year-old man is brought into the ER 4) Prolonged vomiting is associated with what
after a bicycling accident. A car door electrolyte abnormality?
suddenly opened in front of him, of which he a) Hypochloremic hypokalemic metabolic
smashed into and was thrown 15 feet. On acidosis
examination, he is drowsy and confused. He b) Hypochloremic hypokalemic metabolic
opens his eyes when his name is called. He alkalosis
mumbles words that you understand but the c) Hyperchloremic metabolic acidosis
sentences do not make sense. He moves all d) Hyperkalemia
four limbs but does not respond to any e) None of the above
commands. He is able to pull both hands
away when pinched and squirms when his
sternum is rubbed, making no effort to stop 5) A 37 year-old male arrives at the
you. What is his Glasgow COMA Scale Emergency Department unconscious. He is
score? warm and sweaty. His heart rate is 52 bpm,
a) 10 his BP is 90/60. His pupils are constricted,
b) 11 his eyes are teary, and he is drooling. You
c) 9 assume he is suffering from a toxidrome.
d) 8 What antidote will you give him?
e) 7 a) Flumazenil
b) Naloxone
c) Glucagon
2) Which of the following are not consistent d) Atropine
with primary (spontaneous) bacterial e) Ethanol
peritonitis?
a) Abdominal discomfort and fever
b) Ascitic fluid neutrophil count of> 250x106 6) A 16 year-old girl is brought to hospital by
cells/L her frantic parents after a bee sting. Vitals
c) Ascitic fluid WBC count of >500x106 signs are BP 70/40, RR 30 and laboured,
cells/L HR 140, T 37.5. Which of the following
d) Multiple organisms on culture and would not be an option in her management?
sensitivity of ascitic fluid a) Epinephrine
b) Diphenhydramine
c) Methylprednisolone
3) A 25 year-old known substance abuser is d) Salbutomol
brought to the ED with a suspected e) Atropine
overdose. Which of the following is not
considered a universal antidote?
a) Glucose 7) Tension pneumothorax is best diagnosed
b) Oxygen with:
c) Calcium gluconate a) Stat CT scan
d) Naloxone b) Chest x-ray
e) Thiamine c) Watch and wait
d) Clinical exam
e) None of the above
8) A patient presents with decreased level of
consciousness. On exam he has pinpoint
pupils and is unresponsive. Which of the
following is NOT a likely etiology?
a) Cerebellar infarct
b) TCA overdose
c) Morphine overdose
d) Pons hemorrhage
e) Ethanol overdose

9) A 41 y/o male alcoholic is brought into the


ED by ambulance. He has been seizing for
approximately 20 minutes. Appropriate
actions in the next 5 minutes may include all
of the following EXCEPT:
a) Endotracheal intubation, if unable to stop
the seizure rapidly
b) Administration of IV anticonvulsants
c) Performing a complete set of vital signs
d) Ordering a stat EEG
ANSWERS

1. A 3. C 5. D 7. D 9. D
2. D 4. B 6. E 8. B
Endocrinology


1) A 76 year old man from a nursing home is 3) A 37 year old female is referred to you, an
brought into the emergency department endocrinologist, because she is concerned
after staff notice a decreased level of about her weight. She is obese with a BMI
consciousness. He has Type II diabetes of 35.2. On her last physical, her family
mellitus and is diagnosed with doctor found her LDL level to be extremely
hyperosmolar state. All of the following elevated. Her only medications are
may be features of hyperosmolar state acetaminophen PRN for headache and
EXCEPT: quetiapine, which she has been taking for
a) Hyponatremia the past several years for anxiety and to
b) Positive ketones on urine dipstick help her sleep at night. As you chat with
c) Positive glucose on urine dipstick her, you learn she has never married and
d) Elevated blood glucose level never had any children. In fact, she admits
e) History of decreased fluid intake to having few social supports. Apparently,
she has had poor self esteem since she
was young because of unwanted hair
2) All of the following are complications of growth and obesity. She agrees that she
diabetes mellitus EXCEPT: uses food for comfort. Which of the
a) Retinal detachment following statements apply to this case?
b) Gangrene in the feet a) She is at risk for osteoarthritis in the
c) Calcification of cartilage future
d) Abdominal bloating b) You should order gonadotropin levels in
e) Urinary retention her
c) She should be referred to a dietician
d) She should be started on cholesterol
lowering medications immediately
e) With such a high BMI, she should be
initiated on orlistat (pancreatic lipase
inhibitor) immediately

4) A patient complains of a non-tender mass


over the thyroid region on the left side of
her neck. Concerned about a thyroid
disorder, you order the appropriate
investigations. The results are as follows:
TSH: 6.0
Free T4: 20.2
Thyroid antibodies: none
RAIU: No “hot” spots seen

The next investigation(s) you choose to


do are:
a) Watch and wait for 3-6 months
b) FNA
c) Surgical biopsy
d) Trial of L-thyroxine therapy for 6 months
e) None of the above
5) An 8 year-old boy is brought to the office 8) A 58 year-old man with a past history of a
because his mother is concerned he is parathyroidectomy for primary
entering puberty already. You examine him hyperparathyroidism is now in your office
and note the beginnings of facial hair, complaining of headaches worse in the AM
axillary hair and Tanner stage 2 external (made worse by a small MVA he credits to
genitalia. Choose the set of investigations a loss of peripheral vision). You plan to:
you initially want to do: a) Send to the Emergency Department for
a) CBC, lytes, testosterone, bone age, CT an immediate CT head
head b) Check his calcium to ensure there’s no
b) FSH, LH, testosterone, lytes, bone age, remaining parathyroid tissue
DHEA-S c) Check for a pheochromocytoma (which
c) FSH, LH, testosterone, cortisol, DHEA- you know causes H/As) because you
S, 11-OH progesterone, bone age are concerned he has MEN I syndrome
d) Lytes, testosterone, DHEA-S, 17-OH d) Check for a homonymous hemianopia
progesterone, cortisol, bone age because you are worried about a
pituitary tumor
e) Check for a bitemporal hemianopia
6) In the course of DKA, serum potassium because you are worried about a
levels: pituitary tumour
a) Remain unaffected
b) Can appear normal but total body
potassium may actually be low 9) In the treatment of Type I Diabetes, which
c) Can appear normal but total body of the following is true?
potassium may actually be high a) Sulfonylureas are useful as an
d) Will naturally be corrected by insulin adjunctive therapy to insulin
administration b) Most patients are adequately controlled
e) None of the above with one type of insulin (non-mixed) only
c) Once diagnosed with Type I, patients
must immediately be assessed for
7) The “triple bolus” test of pituitary function retinopathy
works by a rapid succession of IV d) During periods of illness or infection,
constituents as follows: patients may require additional insulin
a) insulin – hypoglycemia mediated rise in e) The most common initial presentation is
GH and ACTH visual disturbance
GHRH – rise in LH and FSH
TRH – rise in TSH and PRL
b) CRH – rise in GH and ACTH
GHRH – rise in LH and FSH
TRH – rise in TSH and PRL
c) estrogen – rise in LH, drop in FSH and
PRL
insulin – rise in GH and ACTH
TRH – TSH
d) cortrosyn – rise in GH and ACTH
GHRH – rise in LH and FSH
TRH – rise in TSH and PRL
10) In the diagnosis and treatment of
dyslipidemia, which of the following is
false?
a) Men over age 40 and women over age
50 should be screened to obtain a
fasting lipid profile
b) Treatment with a statin is the treatment
of choice for patients with elevated LDL
c) Treatment with niacin or fibrate is the
treatment of choice for patients with
primarily elevated triglyceride level
d) Patients identified with dyslipidemia
should all meet the target LDL < 2.5
mmol/L and target TC/LDL < 4.0
e) A baseline CK before the start of statin
treatment should be obtained, and a
patient complaining of myalgia and
currently on a statin requires a repeat
CK level
ANSWERS

1. B 3. E 5. D 7. D 9. D
2. C 4. B 6. D 8. E 10. D
Ethical, Legal and Organizational

1) To qualify for federal contributions, 4) Which one of the following would justify a
provincial health insurance plans must patient's being declared incompetent with
guarantee all of the following, EXCEPT: respect to self care?
a) Benefits include all necessary hospital a) The patient has a major mental disorder
care and physicians' services b) The patient has been admitted to
b) Universal coverage on uniform terms hospital
and conditions c) The patient is undergoing
c) Portability of coverage from province to psychotherapy
province d) The patient is unable to understand the
d) Benefits include all necessary out-of- consequences of his/her decisions
hospital drugs, dental care and e) The patient is unable to work because of
prostheses a mental disorder
e) Administration on a non-profit basis

5) Concerning a physician's obligation to


2) Under certain circumstances, it may be provide due care to a patient, which one of
legitimate to detain individuals in an the following is correct?
institution against their wishes, for their own a) Beneficence is a moral commitment but
good and to protect others. This situation not a legal obligation to the well-being of
may arise when: the patient
a) A person has bizarre fantasies and b) Respecting autonomy is both a moral
actions commitment and a legal obligation to the
b) Persons are mentally incompetent and patient
unable to manage their own affairs c) A physician can be found guilty of
c) A close relative submits a petition that negligence even if he/she has acted in
an individual is actually and presently accordance with a practice acceptable
insane to a responsible body of peers
d) The individual's continued liberty poses d) Unanimity of practice is expected of
a danger to themselves and/or to others physicians
e) A person has visual and aural e) Where patient outcomes are poor,
hallucinations physicians may have their practices
scrutinized by the courts but not by peer
organizations
3) In which one of the following situations
would it be unethical for the psychiatrist to
break confidentiality and notify the 6) Which of the following is not an element of
authorities? valid consent?
a) The psychiatrist believes that the patient a) The patient arrives at his/her decision
will probably commit murder voluntarily and without coercion
b) The psychiatrist believes that the patient b) The patient is able to understand the
is actively suicidal information and appreciate the
c) The patient has severely impaired consequences of the decision
judgement and is in a responsible c) Consent is obtained in a written record
position, e.g. an airline pilot d) The consent is specific to the proposed
d) The patient has aggressive fantasies. procedure and to the provider(s) of care
e) The patient has taken a significant e) All information regarding the proposed
overdose intervention, including very remote or
unlikely risks of the intervention, have
been discussed
7) You want to embark on a clinical trial of a 10) Which one of the following is NOT a
new drug for your research project and the necessary element of legally valid consent?
manufacturer of this drug has offered to a) Consent must be given voluntarily by the
fund your trial. Which of the following patient/patient’s agent
violates an ethical principle for human b) The consenting individual must be
research in this situation? deemed capable to give consent
a) Obtaining informed consent personally c) Consent must be specific to the
from your patients proposed intervention
b) Arranging for adverse effects and data d) Consent must be written and signed by
safety to be monitored by the drug the patient/patient’s agent
manufacturer e) The patient/agent must be reasonably
c) Authoring the trial results informed of the risks and benefits of the
d) Writing the study proposal for your proposed intervention
hospital's Research Ethics Board
e) Disclosing the source of funding and the
nature of your participation in this
research to prospective patients

8) All of the following regarding substitute


decision makers are false, EXCEPT:
a) A physician can ask for a substitute
decision maker to make a decision for a
capable patient
b) Substitute decision makers may override
wishes that the patient previously
expressed while still capable
c) Substitute decision makers need to be a
relative of the patient for whom the
decision is being made
d) A physician can appeal to a Consent
and Capacity Board if s/he feels the
substitute decision maker is not acting in
the patient's best interests
e) Informed consent cannot be obtained
from a substitute decision maker

9) Which of the following circumstances is not


automatically reported to the appropriate
authority?
a) Suspected child abuse
b) Suspected domestic violence among
adults
c) A death that occurs during pregnancy
d) A case of Chlamydia
e) A driver with new seizure disorder
ANSWERS

1. D 3. D 5. A 7. B 9. B
2. D 4. D 6. C 8. C 10. D
Family Medicine


1) For anxiety, each of the following 5) A patient presents to the office requesting
pathologies should be considered in the assistance with quitting smoking. You are
differential EXCEPT: considering prescribing buproprion.
a) Arrhythmia Contraindications to this prescription include
b) Asthma/COPD all of the following EXCEPT:
c) Drug-induced a) Concurrent use of nicotine replacement
d) Acute renal failure therapy
e) Pheochromocytoma b) Seizure disorder
c) Eating disorder
d) Previous anaphylactic reaction to
2) A poor response to anti-hypertensive buproprion
therapy could be attributed to each of the e) MAOI use in the past 14 days
following EXCEPT:
a) Suboptimal regimen (e.g. insufficient
dose, inappropriate combination) 6) You suspect that a patient may have
b) Poor compliance developed a drinking problem over the last
c) Oral contraceptives few years, so decide to order some
d) Licorice bloodwork. All of the following are in keeping
e) Low sodium diet with your suspicions EXCEPT:
a) Elevated GGT
b) Elevated platelets
3) Which statement is correct regarding fecal c) Elevated AST
occult blood testing? d) Decreased hemoglobin
a) Every 1 to 2 years starting at 40 years e) Increased INR
old for the general population
b) Every 1 to 2 years starting at 50 years
old for the general population 7) The following is TRUE with regards to
c) Every 3 to 5 years starting at 55 years asthma:
old if a family history of colorectal cancer a) Onset typically occurs > age 10
d) Every 5 years starting at 50 years old for b) It is a restrictive airway disease
the general population c) Warm air is a common asthma attack
e) Unnecessary if the patient has ever had precipitator
a colonoscopy d) Symptoms are due to bronchial
inflammation and bronchodilation
e) Chest x-ray is often normal
4) Which of the following is NOT a major risk
factor for coronary artery disease?
a) Smoking 8) Patient SL presents with symptoms typical
b) Hypertension of a common cold. The organism most likely
c) Sedentary Lifestyle to be responsible is:
d) Family History a) Respiratory syncitial virus
e) Diabetes b) Parainfluenza virus
c) Adenovirus
d) Rhinovirus
e) Coxsackie virus
9) Which of the following is NOT a selective 11) Which of the following is NOT TRUE when
serotonin reuptake inhibitor? considering treatment of a patient with
a) Venlafaxine dysuria?
b) Paroxetine a) Risk factors for complicated UTIs include
c) Fluoxetine male sex, indwelling catheter and
d) Sertraline immunosuppresion
e) Citalopram b) Patients should always be treated for
both gonorrhea and chlamydia, even if a
positive result for only one of these is
10) Which of the following IS a diagnostic obtained
criteria for diabetes mellitus? c) Prophylactic antibiotics may be
a) HbA1c greater than 6% considered in a patient with recurrent
b) Fasting blood glucose greater or equal to UTIs (>3 per year)
6.0 mmol/L d) Pregnant women with bacteruria should
c) Random blood glucose greater than 10 not be treated if asymptomatic
mmol/L e) In adulthood, dysuria is more common in
d) Blood glucose 2 hours post OGTT women than men
greater than 10 mmol/L
e) HbA1c greater or equal to 7%
12) Mrs. HA presents to your office for the first
time complaining of headaches. Of the
following symptoms, which is the LEAST
worrisome?
a) Sudden onset of severe headache
b) Headache is accompanied by nausea
and vomiting
c) Headache is present on awakening
d) New onset headache > 50 years of age
e) Headache accompanied by neck
stiffness
ANSWERS

1. D 4. C 7. E 9. A 11. D
2. E 5. A 8. D 10. E 12. E
3. B 6. B
Gastroenterology

1. Which one of the following is not a 5. Which of the following statements


symptom of Vitamin A deficiency? regarding esophageal and stomach
a) Keratomalacia disorders is false?
b) Poor wound healing a) Epigastric pain may be due to MI,
c) Osteomalacia so an ECG should be obtained in
d) Night blindness all elderly patients with epigastric
discomfort
b) H. pylori is a common, treatable
2. Which one of the following is not a cause of PUD
symptom of folic acid deficiency? c) Upper GI bleeding in most
a) Megaloblastic anemia patients stops spontaneously
b) Glossitis d) All patients with GI bleeding
c) Diarrhea should be risk stratified for proper
d) Neuropathy management and disposition
e) None of the above

3. Which of the following clotting factors is


dependent on Vitamin K? 6. Which of the following is/are considered
a) II risk factors for mesenteric ischemia?
b) VII a) Age greater than 50 years
c) IX b) Valvular or atherosclerotic heart
d) X disease
e) All of the above c) Recent MI
d) Critical illness with hypotension
or sepsis
4. Which of the following is not among the e) All of the above
most common deficiencies associated
with intestinal disease?
a) Folate 7. Which is the least common cause of
b) Calcium lower GI bleed in patients ≥ 60 years of
c) Vitamin B12 age?
d) Zinc a) Diverticulosis
e) None of the above b) Ischemic Bowel
c) AVM
d) Carcinoma
e) thrombocytopenia

8. Which of the following procedures


cannot be done with endoscopy?
a) Thermal Coagulation
b) Injection of alcohol, adrenaline, or
sclerosing agents
c) Variceal band ligation
d) Argon plasma coagulation
e) All of the above can be
performed through endoscopy
9. Which of the following provisions may 13. What is the most common benign tumor
be necessary when transfusing blood or of esophagus?
blood products? a) Fibroma
a) IV Calcium Gluconate b) Lipoma
b) Platelets c) Leiomyoma
c) FFP d) None of the above
d) All of the above

14. What is the treatment of choice for


10. When UGI bleed is caused by PUD, esophageal infection with herpes virus?
which of the following correlates best a) Ganciclovir
with rebleeding b) Acyclovir
a) More than one ulcer at c) Ketoconazole
endoscopy d) Fluconazole
b) Nonbleeding visible vessel
c) Adherent clots
d) Spurting bleeding at time of 15. Which of the following tests are helpful
endoscopy in evaluating RLQ pain?
e) Pigmented spot at endoscopy a) WBC count > 10000
b) Urinalysis
11. What is the most common pathogenesis c) B-HCG
of reflux? d) Abdominal CT
a) Hiatus hernia e) All of the above
b) Decreased lower esophageal
sphincter pressure
c) Increased intra-abdominal 16. What is the most specific clue in the
pressure diagnosis of acute appendicitis?
d) Delayed gastric emptying a) Pain over McBurney’s point
e) Transient lower esophageal b) Leukocytosis
sphincter relaxations c) RLQ pain
d) Anorexia

12. Which of the following statements is


wrong about Barrett’s esophagus? 17. What is the most common cause of non-
a) It is a metaplastic process erosive gastritis?
resulting in replacement of a) Lymphocytic gastritis
squamous epithelium with b) Atrophic gastritis
columnar epithelium c) Celiac Sprue
b) Tobacco and alcohol abuse are d) H. pylori
risk factors for the development
of Barrett’s esophagus
c) Adenocarcinoma may arise in 18. What should be suspected when a
Barrett’s esophagus gastric ulcer is refractory to appropriate
d) Highest incidence of Barrett’s medical management?
esophagus is among Blacks. a) Gastric cancer
e) None of the above b) Surreptitious NSAID use
c) Persistent H. pylori infection
d) Acid hypersecretory state
e) All of the above
19. What are the common metastatic sites 24. What is malabsorbed after distal bowel
for gastrinomas? resection?
a) Kidney a) Ca
b) Lung b) Folic acid
c) Bone c) Vitamin B6
d) A + B d) Vitamin B12
e) B + C e) C + D

20. Which of the following statements is 25. Which of the following conditions favor
false? small bowel bacterial growth?
a) Most patients with acute diarrhea a) Diabetic gastroparesis
require antibiotics b) Hypochlorhydia
b) Correction of fluid and electrolyte c) AIDS
imbalance is the first step in the d) Fistula between large and small
treatment of acute diarrhea bowel
c) Antimotility agents should be e) All of the above
used with caution in the treatment
of diarrhea in IBD patients
d) Enteric salmonella infection must 26. Which of the following laboratory
be treated with antibiotics findings is least likely to be found in the
e) None of the above setting of malabsorption?
a) Iron deficiency
b) Ca deficiency
21. Which of the following must be avoided c) B12 deficiency
in patients with celiac sprue? d) Decreased PT
a) Wheat e) None of the above
b) Barley
c) Rye
d) Oats 27. Which of the following is likely in
e) All of the above untreated malabsorption?
a) Steatorrhea
b) Night blindness
22. What diseases are associated with c) Bone pain
celiac sprue? d) Glossitis
a) IDDM e) All of the above
b) Autoimmune thryoiditis
c) Dermatitis herpetiformis
d) IgA deficiency 28. Which of the following is least common
e) All of the above in IBS?
a) Mucus on the stool
b) Bloating
23. What is malabsorbed after proximal c) Sensation of incomplete fecal
bowel resection? evacuation
a) Ca d) Abdominal pain worsens after a
b) Folic acid bowel movment
c) Iron e) Alternating diarrhea and
d) None of the above constipation
e) All of the above
29. Which of the following extraintestinal 33. Which of the following is not a major
manifestations of IBD occurs cause of acute hepatitis?
independently of intestinal disease a) Hepatitis A
activity? b) Epstein-Barr virus
a) Uveitis c) CMV
b) Episcleritis d) Toxin exposure
c) Erythema nodosum e) None of the above
d) Peripheral arthritis (e.g. knee)
e) Primary sclerosing cholangitis
34. Which patients with hepatitis should be
admitted?
30. Which of the following constitutes the a) Coagulopathic patients
ED management of IBD? b) Patients who are actively
a) Analgesia bleeding
b) Sulfasalazine c) Encephalopatic patients
c) Steroids d) Those whose social situation
d) Antidiarrheal agents would make proper care difficult
e) All of the above e) All of the above

31. Which of the following statements is true 35. Which of the following constitutes the
of bowel disorders? mainstays of treatment in hepatic
a) Appendicitis is a common encephalopathy?
condition with unusual a) Supportive care
presentations; therefore, always b) Lactulose
consider appendicitis in a patient c) Neomycin
with abdominal pain d) Low-protein diet
b) A patient with atrial fibrillation and e) All of the above
abdominal pain has mesenteric
ischemia until proven otherwise.
c) IBD can cause complicated rectal 36. Which of the following is a complication
abscesses or fissures which may of chronic liver disease?
require surgical consultation a) Spontaneous bacterial peritonitis
d) Postoperative adhesions, b) Esophageal varices
incarcerated hernias and cancer c) Increase risk of bleeding
are the most common causes of d) Hepatorenal syndrome
small bowel obstruction e) All of the above
e) All of the above

37. Which of the following is not a cause of


32. Which of the following are common acute pancreatitis?
ultrasound findings of cholecystitis? a) Hypercalcemia
a) Presence of gallstones b) Mumps
b) Gallbladder wall thickening > 3 c) Gallstones
mm d) Erythromycin
c) Pericholecystic fluid e) None of the above
d) Common bile duct dilatation > 6
mm
e) All of the above
38. Which of the following is not among the 43. Which of the following is not associated
common laboratory findings in acute with HCV infection?
pancreatitis? a) IV drug abuse
a) Elevated serum amylase b) Poverty
b) Elevated serum lipase c) High risk sexual behaviour
c) Leukocytosis d) Perinatal transmission
d) Hypoglycemia e) None of the above
e) Hypocalcemia

44. What is the leading cause of


39. Which of the following is not a mode of transfusion-related hepatitis?
transmission for HBV? a) Hepatitis A
a) Sexual contact b) Hepatitis B
b) Percutaneous c) Hepatitis C
c) Blood product transfusion d) Hepatitis D
d) Perinatal e) Hepatitis E
e) Fecal-oral

45. Which of the following increase the rate


40. Which of the following is an extrahepatic of progression to cirrhosis in HCV
manifestation of chronic HBV infection? infection?
a) Serum sickness a) Alcohol consumption
b) Glomerulonephritis b) Male sex
c) Aplastic anemia c) Older age at infection
d) Polyarteritis nodosa d) Co-infection with HIV
e) All of the above e) All of the above

41. Interferon is commonly used in the 46. Which of the following drugs can cause
treatment of chronic HBV infection. liver injury via direct toxicity?
Which of the following is a a) Ethanol
contraindication to interferon therapy? b) Acetaminophen
a) Decompensated liver disease c) Methotrexate
b) Autoimmune diseases d) Azathioprine
c) Active alcohol use e) All of the above
d) Pregnancy
e) All of the above
47. Which of the following laboratory tests
are important in cases of
42. Who should receive the hepatitis B acetaminophen toxicity?
vaccine? a) Liver enzymes
a) All infants b) PT_INR
b) Travelers at risk c) BUN/Creatinine
c) After sexual exposure to HBV d) Arterial blood gas
d) Patients with chronic liver e) All of the above
disease not caused by HBV
e) All of the above
48. Which of the following patient groups is 49. What are the most common primary
at higher risk of getting Primary Biliary benign liver tumors?
Cirrhosis? a) Hemangioma
a) Infants b) Adenoma
b) Elderly male c) Fibroma
c) Elderly female d) A + B
d) Middle-age female e) B + C
e) None of the above

50. Which of the following is an accepted


indication for liver transplantation?
a) Acute liver failure
b) Hepatocellular carcinoma
c) HCV liver disease
d) When no acceptable alternative
forms of therapy exist
e) All of the above
ANSWERS

1. C 12. D 23. E 34. E 45. E


2. D 13. C 24. D 35. E 46. E
3. E 14. B 25. E 36. E 47. E
4. E 15. E 26. D 37. E 48. D
5. E 16. A 27. E 38. D 49. D
6. E 17. D 28. D 39. E 50. E
7. E 18. E 29. E 40. E
8. E 19. E 30. E 41. E
9. D 20. A 31 E 42. E
10. D 21. E 32. E 43. D
11. E 22. E 33. E 44. C
General Surgery


1) Which of the following is NOT an indication 7) Which anastamosis is NOT present in a
for urgent operation whippple procedure?
a) Pulsatile abdominal mass, abdominal a) Gastrojejunostomy
pain, and hypotension b) Hepaticojejunostomy
b) Pancreatitis c) Choledochjejunostomy
c) Strangulated Hernia d) Pancreaticojejunostomy
d) Cecal Volvulus

8) Indications for operative intervention rather


2) Which of the following is a BENIGN tumor of than conservative management for arterial
the liver? insufficiency include all the following
a) Adenoma EXCEPT:
b) Angiosarcoma a) Nocturnal limb pain
c) Hepatoblastoma b) Ischemic ulceration
d) Hepatocellular carcinoma c) Absent pulse
d) Ischemic neuropathy
e) Toe gangrene
3) What is NOT part of Charcot's Triad?
a) Fever
b) Shock 9) Surgical indications for diverticula include all
c) Right upper quadrant pain of the following EXCEPT:
d) Jaundice a) Peritonitis
b) Persisting hemorrhage
c) Fistula
4) Which is an indication for operation in d) Greater than two severe attacks
Diverticulitis e) Palpable abdominal mass in left lower
a) Hinchey Stage 3 quadrant
b) Previous appendiceal phlegmon
c) After 1 attack in an otherwise healthy
individual 10) Six days following a hemicolectomy for
d) Patient preference colorectal carcinoma, a 54 year-old woman
experiences calf pain unilaterally. On
physical examination, the affected side has
5) What is NOT on the differential diagnosis of a greater diameter than the other, and there
acute appendicitis? is tenderness to palpation. Which of the
a) Ovarian torsion following investigations would be the next
b) Dicerticulosis step?
c) Crohns disease a) Venogram
d) Ulcerating colon cancer b) Compression venous Doppler flow
studies
c) X-ray of the affected calf
6) Which of the following neoplasms is NOT d) Ventilation/perfusion scan
associated with HNPCC? e) Compartment pressure monitoring
a) Gastric
b) Renal
c) Pancreatic
d) Endometrial
11) Which of the following radiologic abdominal 15) Which of the following signs and symptoms
plain film findings are consistent with warrant surgical intervention for patients
mechanical bowel obstruction: with small bowel obstruction?
a) A "step ladder" pattern a) Abdominal tenderness
b) Dilated small bowel loops b) Air-fluid levels on abdominal x-ray
c) Absence of gas in large bowel c) Worsening abdominal pain
d) Rows of small gas accumulations in d) Feculent vomitus
valvulae conniventes (i.e. "string of e) Air in the colon and rectum on abdominal
pearls") x-ray
e) All of the above are true

16) A 59 year-old woman presents to her family


physician with a 3 cm palpable, well
12) After a weekend of heavy drinking, a 28 circumscribed, non-tender breast mass. She
years-old male presents with abdominal first noticed it several months ago and
pain radiating to the back. What is the believes it has increased in size since then.
investigative and prognostic modality of Her mother and maternal aunt were
choice for the suspected diagnosis? diagnosed with breast cancer in their early
a) Abdominal ultrasound 50's. She has no other health complaints. A
b) Abdominal CT subsequent mammogram shows no
c) Abdominal plain film abnormalities. Which of the following is the
d) Endoscopic retrograde next most appropriate step in management?
cholangiopancreatography a) Prophylactic mastectomy
e) Laparoscopy b) Repeat mammogram in 6 months
c) Repeat mammogram in 1 year
d) Core needle biopsy
13) Which of the following is associated with e) Ultrasound
biliary colic?
a) Epigastric pain
b) Rebound tenderness 17) Which of the following is false regarding
c) Jaundice pancreatic pseudocysts?
d) Murphy sign a) It is caused by duct leakage
e) All of the above b) Clinically suspected if persisting pain > 2
weeks following diagnosis of acute
pancreatitis
14) Which of the following is false regarding c) Majority are treated surgically
post operative wound infections? d) Surgical intervention is typically delayed
a) S. aureus is the most common cause to allow pseudocyst to mature
b) Usually present with fever post-op day 3- e) Lacks true epithelium
4
c) Increased likelihood in diabetics
d) Mainly treated with antibiotics
e) Risk increases with length of surgery
18) An overweight, 45 year-old man presents 23) Which of the following accounts for the
with left lower quadrant tenderness and a change in pain pattern for appendicitis?
one week history of abdominal pain, loose a) Parasympathetic  Sympathetic
non-bloody stools and worsening fever. b) Sympathetic  Parasympathetic
Laboratory investigation yields leukocytosis c) Somatic  Visceral
with neutrophilia and left shift. What is the d) Visceral  Somatic
diagnostic modality of choice? e) None of the above
a) CT scan
b) Barium enema
c) Ultrasound 24) Which of the following statements is not true
d) Plain abdominal film regarding inguinal hernias:
e) Angiography a) A direct inguinal hernia occurs due to a
weakness in the transversalis fascia
b) Cooper’s ligament is an important
19) Risk factors for hepatocellular carcinoma surgical landmark in the repair of an
include all of the following EXCEPT: inguinal hernia
a) Hepatitis A c) Direct inguinal hernias are more
b) Cirrhosis common in boys
c) Exogenous steroid use d) A pantaloon hernia is a combined direct
d) Hemochromatosis and indirect inguinal hernia
e) Smoking e) An inguinal hernia involving the appendix
is known as an Amyand’s hernia

20) Which of the following is NOT a cause of


major lower gastrointestinal hemorrhage? 25) Which of the following is not true regarding
a) Diverticulitis thyroid carcinoma?
b) Angiodysplasia a) Family history and/or history of radiation
c) Aortoenteric fistula to the neck are indications for immediate
d) None of the above excision
b) These nodules are typically hot on iodine
scan
21) Of the following, the greatest risk factor for c) Symptoms may include voice changes or
breast cancer is? dysphagia
a) Age (>40 y.o.) d) Papillary is the most common subtype
b) >5 years HRT
c) Menarche < 12 y.o.
d) Nulliparity 26) All of the following are manifestations of
portal hypertension except:
a) Splenomegaly
22) Which of the following is not a characteristic b) Esophageal varices
suggestive of melanoma? c) Hemorrhoids
a) Diameter >6mm d) Caput medusae
b) Lack of symmetry e) Bowel ischemia
c) Homogeneous colour
d) Irregular borders
e) Surface elevations
ANSWERS

1. B 7. C 12. B 17. C 22. C


2. A 8. C 13. A 18. A 23. D
3. B 9. E 14. D 19. A 24. C
4. A 10. B 15. C 20. A 25. B
5. B 11. C 16. D 21. A 26. E
6. C
Geriatrics

1) The cause of the greatest decrease in 5) Consequences of immobility include which


Quality of life in the elderly is: ONE of the following:
a) Arthritis a) Diarrhea
b) Dementia b) Maintenance of muscle mass
c) Heart disease c) Urinary retention
d) Stroke d) Pneumonia
e) Hypertension e) Hastened wound healing

2) Which of the following is NOT an agerelated 6) Which of the following is FALSE in terms of
change? falls in the elderly?
a) Impaired myocardial diastolic dysfunction a) They are the most common cause of
b) Increased gastric acid secretion mortality due to injury
c) Decreased drug clearance b) Environment plays a significant role
d) Increased nocturnal sodium and fluid c) Fractures most commonly involve the
excretion humerus
e) Decreased baroreflex sensitivity d) Age-related sensory changes make the
elderly more susceptible
e) Fear of falling contributes to self-
3) Regarding the elderly patient, which of the protection immobility
following apply?
a) Vague symptoms
b) Atypical presentations
c) Loss of function
d) Polypharmacy
e) All of the above

4) An 80 year-old female with a 25% reduction


in her GFR requires which of the following
adjustments for a drug that is predominantly
cleared by the kidneys?
1. 25% decrease in individual doses
2. 25% increase in dosing intervals
3. Increase overall fluid intake
4. Decrease length of therapy by 25%

a) All of the above


b) 1 and 3
c) 2 and 4
d) 2,3, and 4
e) 1 and 2
ANSWERS

1. E 3. E 5. D
2. B 4. E 6. C
Gynecology


1) A 62 year old female presents with post- 6) Which of the following is a correct statement
menopausal bleeding. The most important regarding vaginal discharge:
diagnosis to rule out is: a) Candidiasis is characterized by “cottage-
a) Atrophic vulvovaginitis cheese-like” vaginal discharge and KOH
b) Foreign body wetmount would reveal hyphae and
c) Vulvar intraepithelial neoplasia spores
d) Endometrial cancer b) Bacterial Vaginosis is characterized by
white, thick discharge and a negative
KOH whiff test
2) Contraindications to hormone replacement c) Trichomoniasis is characterized by grey,
therapy include: thin and diffuse discharge and saline
a) Known breast cancer wetmount would reveal hyphae
b) Acute liver disease d) All of the above are false
c) Thromboembolic disease
d) Undiagnosed vaginal bleeding
e) All of the above 7) Which of the following is a correct statement
regarding the Menstrual Cycle:
a) The proliferative/follicular phase of the
3) Which of the following can NOT be used as menstrual cycle is of fixed duration
emergency contraception: b) The secretory/luteal phase of the
a) Post-contraception copper IUD menstrual cycle is of fixed duration, due
b) Post-contraception MirenaTM IUD to the finite lifespan of the corpus luteum
c) Yuzpe method c) Progesterone is the main hormone of the
d) "Plan B" proliferative phase, stimulating rapid
growth of endometrial glands and stroma
d) Estrogen is the main hormone of the
4) A 22 year old G0PO presents to her secretory phase
gynaecologist’s office complaining of
amenorrhea. The most important diagnosis
to rule out is: 8) The most common site of occurrence of
a) Turner’s Syndrome Endometriosis is:
b) Imperforate hymen a) Ovaries
c) Pregnancy b) Rectosigmoid colon
d) Hypothalamic/pituitary dysfunction c) Appendix
d) Broad ligament

5) Which of the following is not a clinical


feature of PCOS: 9) What is the most common causative
a) Hirsutism organism of Pelvic Inflammatory Disease:
b) Infertility a) N. gonorrheae
c) Insulin Resistance b) C. trachomatis
d) Dysmenorrhea c) E. coli
d) Actinomyces israelii
10) A 36 year-old female presents with a 14) Which of the following statements about
mucopurulent vaginal discharge. Gram stain estrogen therapy in postmenopausal women
of a cervical swab shows gram negative is correct?
diploccocci. Which of the following is a) It is a major risk factor for breast cancer
correct? b) It decreases hepatic triglyceride
a) If untreated this condition will likely production
resolve spontaneously c) It produces hypertension
b) The most likely diagnosis is Strep group d) It protects against vertebral compression
B fractures
c) Appropriate treatment is Metronidazole e) It directly stimulates the proliferation of
vaginal cream vascular endothelium
d) Appropriate treatment is a single I.M.
dose of 250 mg of Ceftriaxone
e) Public Health will most likely need to be 15) A couple are unable to conceive after 1
notified year. Of the following statistics regarding
infertility, which are correct:
11) A one year-old female presents with vaginal a) 10-15% of couples are infertile
bleeding. Vaginal inspection reveals the b) Only 40% of couples achieve pregnancy
presence of a multicystic grape-like lesion. within 6 months of trying
The most likely diagnosis is? c) Only 20% achieve pregnancy within 1
a) Sexual abuse year of trying
b) DES syndrome d) Male factors are responsible for over half
c) Sarcoma botyroides of infertility causes
d) Clear cell Adenocarcinoma e) Infertility is labelled after failure to
e) Exposure to exogenous estrogen conceive within 2 years of trying

12) A 24 year-old female presents with 16) Vaginal discharge which is fishy in odor and
abdominal pain. Beta-HCG is negative. associated with >20% clue cells on
Pelvic ultrasound shows a 5 cm right microscopy will not be associated with which
ovarian cyst. You would: of the following:
a) Perform immediate laporotomy a) Be due to gardnerella vaginalis
b) Perform immediate laproscopy overgrowth
c) Aspirate the cyst under ultrasonographic b) Can be treated with metronidazole or
guidance clindamycin
d) Order a CBC and a CA125 c) With the addition of KOH may see
e) Expectant management with repeat hyphae or spores
ultrasound in 8 weeks d) Must be treated in all pregnant women,
including asymptomatic
e) Is rarely associated with inflamed or itchy
13) The commonest type of bleeding vulva
encountered with uterine leiomyomata is:
a) Post-coital spotting
b) Mid-cycle bleeding 17) A 55 year-old woman comes to the office to
c) Hypermenorrhea discuss menopause. You tell her which of
d) Oligomenorrhea the following are a part of this stage:
e) Post-coital staining a) Hot flushes
b) Vaginal dryness
c) Mood swings
d) Insomnia
e) All of the above
18) All of the following features have been 23) A 35 year-old G0P0 has severe pain during
described in women with androgen menses which radiates into the anal region.
insensitivity syndrome EXCEPT: On bimanual exam, she has a small,
a) Mammary aplasia retroverted uterus and tender nodules can
b) Female phenotype be felt on palpation of the uterosacral
c) Absence of wolffian duct structures ligaments. Which of the following conditions
d) Short vagina does she most likely have:
e) Absence of mullerian duct structures a) Chronic PID
b) Adenomyosis
19) On colposcopy, the transformation zone: c) Fibroids
a) Contains columnar epithelium d) Endometriosis
b) Is the area between original squamous e) Uterine carcinoma
epithelium and columnar epithelium
c) Contains metaplastic epithelium
d) Contains atypical blood vessels 24) Which of the following statements is
correct?
20) Regarding laparoscopy: a) Cervical carcinoma is the most common
a) It is contraindicated in patients who are gynecological malignancy in Canada
menstruating b) Lichen sclerosis is treated with estrogen
b) Patients with intestinal obstruction are cream
still able to undergo laparoscopy c) Colposcopy is required after a finding of
c) The most common indication for mild cervical dysplasia on a routine Pap
therapeutic laparoscopy is tubal smear
sterilization d) Follicular cysts typically regress with the
d) All of the above are incorrect following cycle
e) Leiomyomata are malignant tumours
21) One cause of secondary amenorrhea is:
a) Turner syndrome
b) Anorexia 25) Which of the following statements regarding
c) Androgen insensitivity syndrome malignant cervical lesions is true?
d) Gonadal dysgenesis a) 95% are squamous cell carcinoma
e) Imperforate hymen b) CA-125 levels to monitor treatment
effectiveness are indicated
22) Risk factors for cervical carcinoma includes c) The majority of lesions arise outside the
all of the following EXCEPT: transformation zone of the cervix
a) Human papilloma virus type 16 and 18 d) They are not associated with HPV
b) Early age at first intercourse infection
c) Smoking e) Treatment for stage 4 disease is radical
d) High socio-economic status hysterectomy with chemotherapy
e) Multiple sexual partners

26) In endometriosis, the most common location


for disease is:
a) Broad ligament
b) Cul-de-sac
c) Ovaries
d) Appendix
e) Uterosacral ligament
27) All of the following increase the risk of 31) The appropriate next step in a woman found
developing ovarian cancer EXCEPT: to have 1 abnormal Pap smear showing
a) Family history ASCUS is:
b) BCP a) Repeat in 1 year's time
c) Nulliparity b) Repeat in 4-6 months time with
d) Late menopause colposcopy if abnormal
e) Caucasian c) Send patient directly for colposcopy and
LEEP excision
d) Send for colposcopy after 1 abnormal
28) A young female enters your office wanting to Pap
start oral contraceptive pills. You remind e) Repeat Pap immediately and treat for
yourself of the reasons she may not start the HPV infection
pill which are:
a) Impaired liver function
b) Undiagnosed abnormal uterine bleeding 32) A 41 year-old woman presents with obesity,
c) Congenital hyperlipidemia hirsutism and oligomenorrhea, which of the
d) Past history thrombophlebitis following tests will give the least useful
e) All of the above information?
a) Prolactin
b) LH/FSH ratio
29) Which of the following statements regarding c) Free testosterone
endometriosis is incorrect? d) TSH
a) Theories proposed to explain the e) 24-hour urine free cortisol
histogenesis of endometriosis include
retrograde menstruation and coelemic
metaplasia 33) With respect to androgen insensitivity
b) CA-125 is used to make the diagnosis of syndrome, which of the following is true?
endometriosis a) Genotype is XY
c) Symptoms of endometriosis may include b) Breast development and uterus are
urinary frequency and dyschezia absent
d) The differential diagnosis of c) Serum testosterone is below normal
endometriomas include hemorrhagic male range
corpus luteum cysts as well as d) Secondary sex characteristics are male
neoplasms e) The condition is autosomal recessive
e) The risk of endometriosis is several fold
greater if there is a first degree relative
with this condition 34) The new selective estrogen receptor
modulators have all of the following benefits
EXCEPT:
30) A 55 year-old female presents with a a) No agonistic effect on breasts
"period-like" vaginal bleeding for 5 days that b) Provides estrogenic benefit on bone
stopped spontaneously. Your approach to c) Minimal effect on triglycerides and
the problem would be to: cholesterol
a) Reassure the patient and manage d) No agonistic effect on uterine tissue
expectantly e) Provides relief of hot flushes associated
b) Administer provera 10 mg od for 15 days with menopause
c) Administer cyclic estrogen plus
progesterone
d) Obtain endometrial tissue
e) Perform a pap smear plus endocervical
curretage
35) Regarding birth control, which of the 39) Which of the following disease and
following is false? treatment is correctly matched:
a) It is contraindicated in women over age a) HSV and cryotherapy weekly until
35 who are smokers lesions disappear
b) Its efficacy may be decreased when b) HPV and acyclovir 400mg po tid x 7d
taking antibiotics c) Primary Syphilis and Benzathin Penicillin
c) Reduces the risk of ovarian carcinoma G 2.4 million units IM single dose
d) Is associated with only a 10% failure rate d) Inpatient PID and Flagyl 500 mg po BID
e) Can be associated with nausea and x 14d
mastalgia e) Bacterial Vaginosis and Clotrimazole 3d
cream

36) All of the following are contraindications for


the IUD EXCEPT: 40) HRT should be used for the treatment of
a) Uterine anomaly osteoporosis.
b) Pregnancy a) True
c) Cervical infection b) False
d) Fibroids 

e) Undiagnosed vaginal bleeding 


37) All of the following are types contraceptives


that release some type of hormone
(estrogen, progesterone or both) EXCEPT:
a) Mirena
b) Nuvaring
c) Orthoevra
d) Alesse
e) Novo-T

38) All of the following are associated with


PCOS EXCEPT:
a) Obesity
b) Hirsutism
c) Infertility
d) Hypertension
e) Oligomenorrhea
ANSWERS

1. D 9. B 17. E 25. A 33. A


2. E 10. E 18. A 26. C 34. E
3. B 11. C 19. C 27. B 35. D
4. C 12. E 20. C 28. E 36. D
5. D 13. C 21. B 29. B 37. E
6. A 14. D 22. D 30. D 38. D
7. B 15. A 23. D 31. B 39. C
8. A 16. C 24. D 32. B 40. B (false)
Hematology

1. An elevated level of hemoglobin A2 in a 6. Which finding is NOT frequently found in


patient with mild microcytic anemia Chronic Myelogenous Leukemia (CML)?
suggests the diagnosis of: a) Elevated WBCs
a) Alpha-thalassemia b) Elevated vitamin B12 level
b) Sickle trait c) Elevated LDH
c) Beta-thalassemia d) Translocation between
d) Hereditary spherocytosis chromosomes 9 and 14
e) Hereditary persistence of fetal e) Increased uric acid level
hemoglobin

7. Schistocytes on blood film examination are


2. It is unlikely to see macrocytosis in a patient UNLIKELY to be seen in which of the
with anemia in which of the following? following:
a) Reticulocytosis a) Thrombotic thrombocytopenia
b) Vitamin B12 deficiency purpura (TTP)
c) Folate deficiency b) Thalassemia
d) Myelodysplastic syndrome c) Vasculitis
e) Sideroblastic anemia d) Disseminated intravascular
coagulation (DIC)
e) Glomerulonephritis
3. Hemolytic anemia is characterized by all of
the following EXCEPT:
a) Increased LDH 8. All of the following can INHIBIT the
b) Increased reticulocytosis absorption of ingested non-heme iron
c) Increased unconjugated bilirubin EXCEPT:
d) Increased haptoglobin a) Alcohol
e) Lead poisoning b) Achlorhydria
c) Phosphate (i.e. as found in milk)
d) Phytates (i.e. as found in cereals)
4. The treatment of choice for thrombotic e) Antacids
events in the antiphospholipid antibody
syndrome is:
a) Intravenous steroids 9. A 26 year old female presents to emergency
b) High-dose oral steroids with a rapid with widespread petechiae on her arms and
taper legs. Her past medical history is significant
c) Penicillamine for thyroid disease. Her lab values show
d) Aspirin hemoglobin of 115, WBC of 6.6 and platelet
e) Warfarin count of 9. INR and PTT are normal. What is
the most likely diagnosis?
a) ITP
5. All of the following are vitamin-K dependent b) TTP
proteins EXCEPT: c) HUS
a) Protein C d) VWD
b) Antithrombin III e) Iron deficiency anemia
c) Factor IX
d) Factor II
e) Factor VII
10. Which of the following is not associated 13. Which of the following occurs in about 90%
with elevated INR? of patients with chronic myeloid leukemia?
a) Liver disease a) Splenomegaly
b) Warfarin use b) Weight loss
c) Factor VII deficiency c) Hepatomegaly
d) Vitamin K deficiency d) Abdominal pain
e) Factor XII deficiency e) Fever

11.Which of the following is not a feature of 14. What is the most common type of
multiple myeloma? Hodgkin’s lymphoma?
a) Anemia a) Mixed cellularity
b) Hypercalcemia b) Nodular sclerosis
c) Fractures c) Lymphocyte depleted
d) Renal failure d) Lymphocyte predominant
e) Splenomegaly

12. Which of the following is not associated


with thrombotic thrombocytopenic purpura?
a) Fever
b) Elevated indirect bilirubin
c) Increased creatinine
d) Mental status changes
e) Rouleaux on blood film
ANSWERS

1. C 5. B 9. A 13. A
2. E 6. D 10. E 14. B
3. D 7. B 11. E
4. E 8. A 12. E
Infectious Diseases


1) The various species of Campylobacter can 6) Entamoeba histolytica is transmitted to
cause diseases ranging from acute enteritis humans by:
to bacteremia. Which of the following a) Ingestion of infective eggs
modes of transmission does NOT apply to b) Ingestion of cysts
Campylobacter? c) Ingestion of animal tissue that contains
a) Contact with infected animals the larva
b) Contaminated food and water d) Penetration of the skin by infective larva
c) Improperly cooked poultry e) Ingestion of adult form
d) Aerolized droplets
e) Person to person spread via fecal-oral
route 7) Strongyloides spp. is transmitted to humans
by:
a) Ingestion of infective eggs
2) Which of the following is NOT a common b) Ingestion of cysts
infectious cause of acute diarrhea? c) Ingestion of animal tissue that contains
a) Escherichia coli the larva
b) Shigella d) Penetration of the skin by infective larva
c) Norwalk virus e) Ingestion of adult form
d) Vibrio cholerae
e) Helicobacter pylori
8) The biosynthesis of fungal ergosterol is
inhibited by:
3) A virus that is not inactivated by mild a) Amphotericin B
detergents that solubilize phospholipid b) Griseofulvin
membranes is: c) Flucytosine
a) Poliovirus d) Nystatin
b) Variola virus e) Ketoconazole
c) Cowpox virus
d) Vaccinia virus
9) At what CD4 count are HIV patients at
increased risk of developing PCP?
4) All the following are true statements about a) CD4 count >500x106
viruses EXCEPT: b) CD4 count 200-499x106
a) They are obligate intracellular parasites c) CD4 count <200x106
b) They are filterable agents d) CD4 count < 500x106
c) They are simply organized e) None of the above
d) They are devoid of enzymes
e) They may contain double stranded DNA
10) Which of the following is not a complication
of meningitis?
5) A viral genome that does not replicate in a) Hyponatremia
the cytoplasm of the infected cell is: b) Seizure
a) Poliovirus c) Deafness
b) Rabies virus d) Hydrocephalus
c) Cytomegalovirus e) None of the above
d) Rubella virus
e) Mumps virus
11) Which of the following does not cover
pseudomonas?
a) Imipenem
b) Tobramycin
c) Pip-tazo
d) Ceftazidime
e) Ceftriaxone

12) Which of the following ways is TB least


likely to present?
a) Hepatitis
b) Meningitis
c) Osteomyelitis
d) Septic arthritis
e) Peritonitis
ANSWERS

1. D 4. D 7. D 9. C 11. E
2. E 5. C 8. E 10. E 12. D
3. A 6. B
Nephrology


1) What underlying diagnosis is suggested in a 5) What is the most common etiology of
patient with acute renal failure and white nephrotic syndrome among Caucasians?
blood cells or white blood cell casts? a) Membranous nephropathy
a) Interstitial nephritis b) Focal segmental glomerulosclerosis
b) Acute tubular necrosis c) Acute tubular necrosis
c) DIC d) Lupus
d) Nephrotic syndrome e) IgA nephropathy
e) Multiple myeloma f) Post-streptococcal glomerulonephritis

2) What is the etiology of stenosis in renal 6) What is the most common intrinsic renal
artery stenosis? disease that leads to acute renal failure?
a) 2/3 secondary to atherosclerosis, 1/3 a) Acute tubular necrosis
secondary to fibromuscular dysplasia b) Hypovolemia
b) 2/3 secondary to atherosclerosis, 1/3 c) Rhabdomyolysis
secondary to DIC d) Wegener's granulomatosis
c) 1/2 secondary to aortic dissection, 1/2 e) Multiple myeloma
secondary to atherosclerosis
d) 2/3 secondary to fibromuscular
dysplasia, 1/3 secondary to Acute 7) What is the most common etiology of
tubular necrosis nephrotic syndrome in African Americans?
e) 2/3 secondary to fibromuscular a) Focal segmental glomerulosclerosis
dysplasia, 1/3 secondary to b) Membranous nephropathy
atherosclerosis c) Lupus
d) Sarcoidosis
e) IgA nephropathy
3) What illness commonly presents with
arthralgias, purpura, abdominal pain,
microscopic hematuria, mild proteinuria, 8) What is the main cause of End Stage Renal
azotemia, and proliferative Disease?
glomerulonephritis characterized by IgA a) Diabetes
deposits? b) Essential HTN
a) Henoch-Schonlein purpura c) Glomerulonephritis
b) Membranous nephropathy d) Polycystic kidney disease
c) Acute tubular necrosis e) IgA Nephropathy
d) Hyperkalemia
e) Multiple myeloma
9) What is the most common form of idiopathic
glomerulonephritis?
4) Which of the following is not an EKG change a) Post-streptococcal glomerulonephritis
in hyperkalemia? b) Essential HTN
a) Peaked T waves c) Membranous nephropathy
b) Prolonged P-R d) Acute tubular necrosis
c) Wide QRS with bradycardia e) IgA Nephropathy
d) V-fib with asystole
e) U waves
10) A 65 year-old male with back pain, nephrotic 14) Which of the following is true with respect to
syndrome and anemia present to the ER. proteinuria?
Ultrasound shows normal kidney size. His a) All proteinuria is secondary to glomerular
creatinine is 500. Which diagnosis best fits disease > 2 g/24 h = nephrotic syndrome
the scenario? b) Is always abnormal and indicative of
a) Polycystic kidney disease serious renal disease
b) Chronic GN c) It may be normal for an individual to
c) Multiple myeloma have <150 mg per day of proteinuria
d) Diabetic nephropathy d) If a patient has 1.5 g of protein in 24 h
e) Analgesic abuse they must have tubular-interstitial
disease

11) Which of the following are indications for


dialysis in ARF? 15) In acute pyelonephritis, which of the
a) Severe alkalosis unresponsive to following is most commonly associated with
medical therapy bacteremic spread from a distant focus?
b) Severe acidosis unresponsive to medical a) Escherichia coli
therapy b) Proteus sp.
c) Severe hypokalemia unresponsive to c) Staphylococcus aureus
medical therapy d) Serratia sp.
d) Severe hypercalcemia unresponsive to e) Enterococcus sp.
medical therapy
e) b and d
16) A patient presents with a decreased level of
consciousness and visual difficulties. Blood
12) Which of the following is true with respect to work reveals an anion gap of 22 and an
diabetes and kidney disease? osmolar gap of 24. Which of the following is
a) Primarily affects the tubules most likely responsible?
b) Earliest sign is decreased GFR a) Ethanol
c) Microalbuminuria is a late sign of DM b) Salicylates
nephropathy c) Renal tubular acidosis type I
d) Threshold for dialysis is same as other d) Methanol
CRF patients e) Diabetic ketoacidosis
e) BP control slows progression of DM
nephropathy
17) In which of the following disease processes
would you likely see a bland urine sediment
13) Which of the following is least likely to a) Goodpasture’s syndrome
contribute to renal failure in myeloma? b) Post streptococcal glomerulonephritis
a) Hypercalcemia c) Membranoproliferative
b) Amyloidosis glomerulonephritis
c) Infiltration of the kidney by myeloma cells d) Focal segmental glomerulosclerosis
d) Hyperuricemia e) Puaci immune glomerulonephritis
e) Intratubular light chain deposition
ANSWERS

1. A 5. A 9. E 13. C 17. D
2. A 6. A 10. C 14. C
3. E 7. A 11. B 15. E
4. E 8. A 12. E 16. C
Neurology


1) For the management of an acute delirium 5) When examining a patient with a working
acquired in the hospital, which one of the diagnosis of Bell's palsy what is most
following options would be least important too rule out?
appropriate? a) A brainstem lesion
a) Treating the underlying cause b) An idiopathic cause
b) Promptly increasing antibiotic coverage c) An Ebstein-Barr virus infection
c) Antipsychotic medication as needed d) Diabetes melitus mononeuropathy
d) Avoiding foley catheters and physical e) Parotid gland pathology
restraints
e) Providing the patient with a well-lit room
6) Which medication would you expect to
worsen the common symptoms of
2) When differentiating between seizure and Parkinson's Disease?
syncope, which of the following is important a) Bromocriptine
to consider? b) Benztropine
a) Onset c) Lithium
b) EEG d) Amantadine
c) Autonomic features e) Haldol
d) Urinary incontinence
e) All of the above
7) Which of the following interventions would
benefit any patient suffering from
3) In the early stages of Alzheimer's disease myasthenia gravis?
what is the most common deficit to be a) Thymectomy
expected? b) Acetylcholinesterase inhibitors
a) Major depression c) Edrophonium
b) Parkinsonism d) Only a and b
c) Psychosis e) All of a, b, and c
d) Memory impairment for newly acquired
information
e) Memory impairment for events prior to 8) A 30-year old hairdresser is experiencing a
the onset of degeneration headache as you, a medical student about
to graduate, are sitting in the leather chair.
Which of the following questions would be
4) A patient that has just suffered a stroke finds least useful in assessing her complaint?
it difficult to utter words in either English or a) Is there a family history of similar
his native Portuguese. He is not pointing to headaches?
the ceiling when prompted with the aid of a b) Does the pain seem to come from
translator, nor does he respond when asked behind one eye?
to repeat a word he had just managed to c) Did your headache develop suddenly
speak out. In an otherwise functional and unexpectedly?
individual, these deficits can be most likely d) Are you having trouble or discomfort
due to this type of aphasia: chewing gum and/or talking during the
a) Broca's non-fluent aphasia headache?
b) Global non-fluent aphasia e) At What time of day is the headache at
c) Anomic fluent aphasia its worst?
d) Wernicke's fluent aphasia
e) Conduction fluent aphasia
9) When investigating for the cause of a stroke 13) A 25 year-old man is admitted with a history
which one of the following is most likely to suggesting seizures. Which of the following
yield abnormal findings? would not support this diagnosis?
a) An echocardiogram a) Urinary incontinence
b) An EEG b) The sound of voices preceding events
c) An electrocardiogram c) Drowsiness and weakness following the
d) A biopsy from a large artery event
e) A CT of all the long bones d) Rarely occur when recumbent
e) None of the above

10) A 74 year-old, right-handed man presents


with a past medical history of hypertension 14) Which of the following would not be
and dyslipidemia for 30 years. He is a expected in a right-sided Brown-Séquard
retired banker who recently has had trouble syndrome?
calculating his restaurant bill. He also a) Right-sided hemiparesis
notices that his writing has deteriorated. On b) Right-sided decreased proprioception
physical exam, he has difficulty naming his c) Left-sided decreased sensitivity to
fingers and is confused with distinguishing pinprick
left from right. The lesion is most likely in d) Left-sided decreased vibration sense
which part of the brain? e) None of the above
a) Right parietal
b) Left parietal
c) Left temporal 15) Which of the following is true of Myasthenia
d) Right temporal Gravis?
e) Frontal a) In patients older than 60, thymic
hyperplasia is a common etiology
b) Often associated with thyroid disease
11) Which of the following pair of CNS lesions c) Antibodies that are produced against
and corresponding visual field defects is acetylcholinesterase
incorrect? d) Associated with small cell lung
a) Temporal lobe tumour – superior carcinoma
quadrantanopia
b) Frontal lobe tumour – altitudinal field
defect
c) Pituitary tumour – bitemporal
hemianopsia
d) Occipital lobe tumour – homonomous
hemianopsia
e) Multiple Sclerosis – central scotoma

12) A 63 year-old woman develops intermittent


dizziness. Examination discloses diminished
corneal reflex and mild hearing loss in the
right ear. The most likely diagnosis is:
a) Cerebellopontine angle tumour
b) Benign paroxysmal positional vertigo
c) Lateral medullary syndrome
d) Méniére disease
e) None of the above
Neurology CASE 1:
A 45 year-old migrant farm worker comes to see you with sudden onset of severe pain in
his back and right leg after lifting some bales of hay. The pain radiates into the lateral
aspect of the right foot and is associated with numbness along the lateral aspect of the
foot. He has had a 5 year history of lower back pain with a similar episode of pain 1 year
ago which was concentrated at the posterolateral aspect of the right calf and lateral
aspect of the right foot. It became progressivelyworse and then resolved over a 3 week period.

1. What elements of the history would you ask 2. What elements of the physical exam would
about in order to formulate your diagnosis? you focus on? Choose up to five.
Choose up to five. 1. Examination of skin
1. Diffuse muscle cramps 2. Range of motion of lumbar spine
2. Alcohol intake 3. Blood pressure
3. Effect of NSAIDs 4. Inspection of muscle bulk in lower extremities
4. Family history of back problems 5. Sensory exam of arms
5. Previous history of back injury 6. Peripheral pulses
6. Shoulder pain 7. Reflexes in lower extremities
7. Weakness in right leg 8. Power testing of legs
8. Allergies 9. Power testing of arms
9. Knee problems 10. Sensory exam of legs
10. Urinary incontinence 11. Respiratory rate
11. Skin rashes 12. Temperature
12. Previous effective physiotherapy 13. Straight leg raise test
13. Headaches 14. Abdominal exam
14. Nocturia 15. Rectal exam
16. Pulse
17. Range of motion of knee
18. Auscultation of chest
19. Romberg test
ANSWERS

1. B 4. B 7. D 10. B 13. D
2. E 5. A 8. D 11. B 14. D
3. D 6. E 9. C 12. A 15. B

Neurology Case 1
Case 1 #1:
3, 5, 7, 10, 12

Case 1 #2:
7, 8, 10, 11, 14
Neurosurgery

 

1) In taking a history from a patient with a ring-
enhancing lesion seeon CT, all of the 4) Which of the following is not true concerning
following should be considered in your Brown-Sequard syndrome?
differentialdiagnosis EXCEPT: a) Contralateral spinothalamic deficits
a) Breast cancer metastasis b) Ipsilateral spinothalamic deficits
b) Meningioma c) Ipsilateral dorsal column deficits
c) Glioblastoma d) Ipsilateral pyramidal tract deficits
d) Infarct
e) Cerebral Abscess
5) A 52-year-old man presented to the urgent
care center 2 weeks ago with severe left
2) A 75 year old man is brought to your office shoulder pain. He wasdiagnosed with
by his daughter, who is his primary bursitis and treated with NSAIDs. The pain
caregiver. She describes a gradual decline has gradually improved, but the patient has
in her father's gait over the last two months. scheduled an office visit because he is
She now describes his feet as being 'glued' concerned about weakness of the left arm.
or 'magnetized' to the floor. He also has a On examination, the patient has full passive
worsening dementia and has recently range of motion of the arm and shoulder
become incontinent of urine. Assuming this without pain. Marked atrophy and weakness
patient has had no prior investigations, are noted in the left deltoid and shoulder
which of the following is the most girdle muscles. Biceps and triceps reflexes
appropriate next step in his management: are absent. The remainder of the
a) Lumbar puncture examination is unremarkable. Which of the
b) Routine Bloodwork (CBC, lytes, BUN, following is the most likely diagnosis for this
Cr) patient?
c) Urinalysis a) Thoracic outlet syndrome
d) CT or MRI of the head b) Brachial plexitis (Parsonage-Turner
e) Carotid dopplers syndrome)
c) Rotator cuff tear
d) Spinal cord tumour
3) A 40 year old lady is hospitalized for e) Lacunar infarction
treatment of a severe, bilateral pneumonia
with parapneumonic pleural effusions. Two
days into her stay, she develops a severe
headache, then has a seizure overnight.
She has no prior history of epilepsy. What is
your most likely diagnosis:
a) Stroke
b) Cerebral abscess
c) Bacterial endocarditis
d) Migraine
e) Toxoplasmosis secondary to HIV/AIDS
6) A 24-year-old man is brought to the 7) A 46-year-old woman is brought to the
emergency department by the emergency emergency department by EMS after being
medical service (EMS). He suffered head involved in a car accident.She was a
trauma 20 minutes ago while playing passenger in the back seat of the car. The
football. Immediately after the event, he lost accident involved frontal impact, with the car
consciousness for 3 minutes and then woke moving at 50 mph. The patient was not
up mildly confused. He complains of a wearing a seatbelt. The driver says she has
moderate frontal headache. On physical not been awake since the accident, which
examination, the patient's vital signs are occurred 30 minutes ago. On admission, the
stable, his Glasgow Coma Scale (GCS) patient's vital signs are as follows: blood
score is 15, and he has no focal signs on pressure, 100/60 mm Hg; heart rate, 78
neurologic examination. What interventions beats/min; respiratory rate, 8 breaths/min;
would be appropriate in the treatment of this GCS score,7. CT scan shows a frontal
patient? epidural hematoma with mass effect. How
a) Continue with observation and repeat would you treat this patient?
neurologic examinations; repeat a) Intubate the patient, administer
assessment with the GCS periodically; hyperventilation to a carbon dioxide
and consider imaging with a CT scan to tension (PCO2) of 25 to 35 mm Hg,
rule out contusions induce a barbiturate coma, and admit the
b) Continue with observation and repeated patient to the ICU for further evaluation
neurologic examinations; repeat b) Intubate the patient, administer
assessment with the GCS periodically; hyperventilation to a PCO2 of 25 to 35
and obtain an MRI mm Hg, and ask for emergent
c) Admit the patient for prolonged neurosurgery consult for evacuation of
observation; obtain a CT scan to rule out the hematoma
contusions; and start I.V. mannitol for c) Intubate the patient, administer
brain edema hyperventilation to a PCO2 of 25 to 35
d) Admit the patient to the ICU; obtain an mm Hg, admit to ICU for close
MRI; and consider intraventricular observation, and consult neurosurgery
monitoring of intracranial pressure (ICP) for intraventricular ICP monitoring
d) Admit to ICU for further evaluation and
start mannitol and steroids
8) A 22-year-old man is transferred to your 11) In a patient that has been hospitalized for
hospital from a local hospital, where he subarachnoid hemorrhage with evidence of
presented 3 hours ago with closed head vasospasm, which of the following are
trauma. He lost consciousness for 10 appropriate treatment?
minutes. At the first hospital where he was a) Triple H Therapy
taken, he was given pain medications, and a b) Nimodipine
CT scan was performed; the CT scan was c) Angioplasty
negative. The patient is awake and d) a & c
complains only of moderate headache. His e) a, b & c
physical examination is unremarkable. The
family is concerned about the development
of seizures in the future, because they had a 12) Which of the following is not a classic sign of
relative who had that problem. What would a basal skull fracture?
you recommend regarding prophylaxis for a) Battle sign
seizures in this patient? b) Raccoon eyes
a) Phenytoin for 1 to 2 weeks c) Hemotympanum
b) Carbamazepine for 6 months d) Freedman sign
c) Obtain an electroencephalogram; if it is e) CSF rhinnorhea/ottorrhea
abnormal, start phenytoin
d) Do not start any antiseizure medication
at this time 13) A 19 year-old female with a traumatic head
injury is brought to the ED by EMS. She is
9) A 65 year-old gentlemen originally hemodynamically stable but requires
hospitalized for work-up and treatment of a assisted ventilation through an endotracheal
subarachnoid hemorrhage is initially found tube. She does not open her eyes to painful
to have no focal neurological deficits. Six stimuli and maintains an abnormal extension
days after being hospitalized, you are called posture. Which of the following is incorrect?
to his bedside by his nurse as he has a) Her GCS is 3T
become increasingly lethargic, confused, b) A GCS of 8 is an indication for intubation
and has new focal weakness. What is the c) Isolated head injuries can cause shock
likely cause of his deterioration? d) She should be ventilated to a pCO2 of
a) Intra-cerebral hemorrhage 30-35 mm Hg
b) Re-ruptured aneurysm e) All are incorrect
c) Pneumonia
d) Vasospasm
e) Intra-ventricular hemorrhage 14) A 52 year-old dentist comes to your office
complaining of severe bilateral buttock
cramps and thigh fatigue during a tennis
10) In a patient that has been hospitalized for a match and recent onset of impotence. The
subarachnoid hemorrhage, what is the most likely diagnosis is:
appropriate pharmacological prevention for a) Lumbosacral disc problem
vasospasm? b) Multiple sclerosis
a) Metoprolol c) Leriche syndrome
b) LMWH d) Metastatic carcinoma of the spine
c) Triple H Therapy
d) b & c
e) Nimodipine
15) A young woman who was involved in a cliff 19) A 72 year-old man on physical examination
diving accident is brought into the is found to have expressive dysphasia and
emergency department unresponsive to mild right arm weakness. The most probable
deep pain with a right pupil that is dilated location of his lesion is:
and non-reactive. The left pupil is normal. a) Right parietal lobe
The most appropriate initial treatment is: b) Left frontal lobe
a) Endotracheal intubation c) Right frontal lobe
b) CT scan of the head d) Left parietal lobe
c) 500 cc normal saline IV e) Basal ganglia
d) IV steroids
e) IV mannitol
20) Initial management of any patient with coma
of undetermined cause includes all
16) A 78 year-old woman complains of EXCEPT:
experiencing headaches and progressive a) Clear and secure the airway
confusion for the last month. She has a left b) Naloxone
hemianopia and cannot dress herself. A CT c) D50W 50 mL IV
scan demonstrates a large, irregularly d) Dexamethasone 16 mg IV
enhancing mass in the right parietal lobe. e) Thiamine
There is no obvious systemic disease. The
most likely diagnosis is:
a) Brain abscess 21) A 53 year-old male presents to the ED with
b) Glioblastoma multiforme new onset of a severe headache associated
c) Meningioma with nausea and vomiting. There is no
d) Metastasis history of trauma. He is alert and oriented
e) CNS lymphoma with no neck stiffness. Anisocoria is present.
The most likely diagnosis is:
a) Intracranial bleed
17) A patient with a subarachnoid hemorrhage b) Cluster headache
(SAH) caused by a right anterior c) Meningitis
communicating artery aneurysm undergoes d) Migraine headache
successful surgery 2 days after the e) Tension headache
hemorrhage. Three days later, right arm
weakness develops. The most likely
diagnosis is: 22) A 24 year-old woman arrives at the
a) Hydrocephalus Emergency Department unconscious. Her
b) Meningitis BP is 90/60, her heart rate is 60 bpm, she is
c) Repeat hemorrhage breathing at 8 breaths per minute and her
d) Vasospasm O2 sat is 86%. Her eyes remain closed even
e) None of the above after pain stimulation and the only sounds
she makes are incomprehensible. Her
18) A 60 year-old man presents with back and elbows and wrists are flexed with her feet
leg pain and trouble urinating. On extended. This patient’s GCS score is:
examination, he has decreased sensation a) 3
over the buttocks, normal motor power, and b) 4
absent ankle jerks bilaterally. How would c) 5
this patient best be investigated? d) 6
a) Plain films of the lumbar spine e) 7
b) CT of lumbar spine
c) MRI of spine
d) Investigate only if no improvement after
6 weeks of symptomatic treatment
e) Gallium scan of the spine
23) Which physical exam finding below is 26) A neurosurgeon complains of a 3 week
usually not associated with increasing history of awakening at night with right-hand
intracranial pressure? discomfort that resolves after several
a) Deteriorating level of consciousness minutes. On examination, he has mild
b) Increasing heart rate weakness of thumb abduction and
c) Increasing blood pressure diminished pain sensibility on the palmar
d) Yawning, hiccoughing, vomiting aspect of the thumb and index finger. The
e) Unilateral sixth cranial nerve palsy most likely diagnosis is:
a) Carpal tunnel syndrome
b) Cervical radiculopathy
24) A 19 year-old woman is brought to the c) Reflex sympathetic dystrophy
emergency room following involvement in a d) Tendonitis
motor vehicle accident. On examination, she e) Left middle cerebral artery ischemic
has a GCS of 10 and swelling over the attacks
occipital protuberance. The most
appropriate imaging study is:
a) MRI of skull and contents 27) A 73 year-old woman presents with a 6
b) Skull films month history of deteriorating gait and low
c) Head CT back discomfort, exacerbated by walking.
d) Cerebral angiogram Examination is unremarkable except for
e) CT myelogram hypoactive muscle stretch reflexes in the
legs. X-rays of the lumbosacral area shows
the expected degenerative changes
25) An 80 year-old woman suffers cervical associated with a woman of her age. The
spinal soft-tissue injury in a motor vehicle most likely diagnosis is:
accident with no skeletal or neurologic a) Acute lumbar disc hernation
damage documented at the time. Three b) Lumbar stenosis
months later, she presents with sudden c) Myopathy
onset of homonymous right upper d) Normal pressure hydrocephalus
quadrantanopia. CT demonstrates a non- e) Cervical stenosis
hemorrhagic lesion in the left lower occipital
lobe. Which imaging study would likely yield
the most useful information? 28) L4/L5 disc prolapse. What is observed?
a) Carotid Doppler ultrasound a) Loss of ankle-jerk reflex
b) Echocardiography b) Numbness on the lateral side of the foot
c) MR angiography c) Positive femoral nerve stretch test
d) SPECT scan d) Weak quadriceps
e) C-spine plain films e) Foot drop
ANSWERS

1. B 7. B 13. C 19. B 25. C


2. D 8. D 14. C 20. D 26. A
3. B 9. D 15. A 21. A 27. B
4. B 10. E 16. B 22. D 28. E
5. B 11. D 17. D 23. B
6. A 12. D 18. C 24. C
Obstetrics 


No Questions at this time


Ophthalmology


1) Which statement about corneal abrasions is 6) Patients with type II diabetes should be
incorrect? initially seen by an ophthalmologist for a
a) Abrasions are accompanied by dilated fundus exam:
photophobia a) Within 5 years of diagnosis
b) Abrasions are painful b) Within a year of diagnosis
c) The eye is often red c) At 10 years
d) Fluorescein will delineate the abraded d) When vision changes
area e) At the time of diagnosis
e) Abrasions often cause scarring

7) Which of the following is false concerning


2) A patient complains of sudden onset herpes zoster ophthalmicus:
flickering lights and a persistent loss of part a) It is more common in elderly and
of the visual field but no pain. This patient immunocompromised patients
likely has: b) Vesicles on the tip of the nose suggest
a) Posterior cerebral artery occlusion involvement of first branch of the fifth
b) Retinal detachment cranial nerve
c) Migraine c) Corneal sensation may be reduced on
d) Optic neuritis testing
d) Early treatment with an antiretroviral
such as Acyclovir is of limited benefit
3) The most common cause of an ISOLATED e) Referral to ophthalmology should be
third cranial nerve palsy with pupil made if ocular involvement is suspected
involvement is:
a) Vascular hypertension
b) Multiple sclerosis 8) Which of the following concerning a
c) Diabetes mellitus chalazion is false?
d) An aneurysm a) Systemic antibiotics are a useful initial
e) A neoplasm treatment
b) It is a chronic granulomatous
inflammation of the meibomian gland
4) A patient with acute angle closure glaucoma c) Hot compresses are of value as initial
can present with all of the following treatment
EXCEPT: d) A recurrent lid mass should be biopsied
a) A fixed, dilated pupil e) Initial treatment with topical antibiotics
b) Nausea and vomiting may be beneficial
c) Complaint of haloes around objects
d) Ocular hypotension
e) A steamy cornea on ocular exam 9) If multiple vertical epithelial lesions are seen
on slit lamp exam, one should suspect:
a) Herpes zoster ophthalmicus
5) Signs of diabetic retinopathy include all of b) Herpes simplex keratitis
the following EXCEPT: c) Foreign body behind the upper lid
a) Dot and blot hemmorhages d) A chalazion
b) Hard exudates e) Dye eye syndrome
c) Venous beading
d) Macroaneurysms
e) Macular Edema
10) Which is not a major risk factor for open 12) A 55-year-old male patient presents to the
angle glaucoma? ER with sudden painless loss of vision in his
a) Black race right eye. The patient’s medical history
b) Family history of glaucoma consists of Rheumatoid Arthritis currently
c) Intraocular hypertension >21 mm Hg well controlled on Plaquenil and poorly
d) Hyperthyroidism controlled Hypertension. The patient says
e) Age that he had sudden loss of vision in his right
eye 5 days ago and that the vision has not
recovered since. On ocular examination,
11) A 57-year-old female patient presents with a you discover a normal anterior segment
6-day history of new onset headache that is while fundoscopy reveals widespread
moderate-severe in nature. She says that hemorrhages and areas of infarction
the headache is unilateral and she has also throughout the retina. What is the most likely
experienced a blurring of her vision during diagnosis?
this time. Upon further questioning she also a) The patient has likely developed uveitis
complains of some pain in her jaw when secondary to his diagnosis of
eating. On examination, you find a pale optic Rheumatoid Arthritis
nerve in her left eye. What is the next step in b) Bulls-eye maculopathy caused by
this patient’s management? Plaquenil
a) Reassure the patient that she is c) Retinal Detachment
experience migraine headaches and d) Central Retinal Vein Occlusion
advise her to follow-up after 2 weeks
with a headache diary
b) Order bloodwork for ESR and CRP and 13) An 84-year-old female patient presents with
treat with oral Prednisone if these are a 2-year history of gradually worsening
elevated vision. She continues to see 20/40 in both
c) Treat the patient through surgical eyes. On examination, you see a normal
excision of her temporal artery anterior segment and normal lenses.
d) Obtain a temporal artery biopsy and start Fundoscopy reveals mild drusen in both
the patient on oral prednisone therapy maculae with no hemorrhages visible. What
is the most appropriate next step in the
management of this patient’s condition.
a) Treat with anti-VEGF agents such as
Avastin or Lucentis
b) Place the patient on a regimen of ocular
vitamins such as Vitalux
c) Laser Photocoagulation
d) Photodynamic therapy with Visudyne
14) A 55-year-old man presents with a 18) The causes of efferent pupil defects
complaint of seeing floaters in his vision include… (choose the correct answer)
over the last 1.5 years. There has been a a) Damage to the medial geniculate
gradual increase in the number of these nucleus
floaters but there has not been any sudden b) Damage to the optic nerve
change in his vision. He does not complain c) Adie's syndrome
of seeing any flashes of light and has not d) Damage to the lateral geniculate nucleus
noticed any shadows or curtains over his
vision. There is no photophobia and no
ocular pain. Examination reveals mild 19) A 59-year-old male presents to his family
cataracts in both eyes and his retina appear physician with a 6-month history of
normal and healthy. What is the most likely progressive ptosis and diplopia. On
cause for this patient’s symptoms? examination, the left eye is normal, but the
a) Peripheral Retinal Detachment not right eye exhibits a significant ptosis,
visible with slit lamp examination limitation of all extraocular movements, a
b) Central Retinal Artery Occlusion small pupil not reactive to light, and loss of
c) Posterior Vitreous Detachment corneal sensation. What is the most likely
d) Cataract diagnosis?
a) Graves ophthalmopathy
b) Myasthenia gravis
15) Which lesion would produce a homonymous c) Multiple sclerosis
hemianopia? d) Intercavernous internal carotid artery
a) To the optic tract aneruysm
b) To the optic nerve
c) At the chiasm 20) A 2-year-old girl presents with a white
d) At the retina pupillary reflex (leukocoria) which the child’s
mother noticed in some recent family
photos. What is the least likely diagnosis?
16) In Horner's syndrome a) Retinopathy of prematurity
(choose correct answer) b) Retinoblastoma
a) 1.0% hydroxyamphetamine will dilate a c) Congenital glaucoma
postganglionic lesion d) Cataract
b) 1.0% phenylephrine will dilate a
preganglionic lesion
c) The miosis is most apparent in low light
levels
d) The ptosis only affects the upper lid

17) In afferent pupil defects (choose correct


answer)
a) The pupil is fixed and dilated
b) The pupil responds to light shone into
the second, normal eye
c) Both pupils constrict when light passes
from the normal to the abnormal eye
d) The damage could be to the oculomotor
nerve
ANSWERS

1. E 5. D 9. C 13. B 17. B
2. B 6. E 10. D 14. C 18. C
3. D 7. D 11. D 15. A 19. D
4. D 8. A 12. D 16. C 20. C
Orthopedics

1) A 17 year old male was skateboarding and 3) A 45 year old female is picking apples from
fell while attempting to jump a flight of a tree when she slips and falls from a
stairs. He lands with his upper arm against height of 14 feet, landing on her feet. Her
the edge of a step and hears a crack. At right ankle is very sore after the injury so
the hospital he is told that he has suffered she proceeds to the local emergency room.
a fracture of the left humeral shaft. What According to the Ottawa Ankle Rules, which
nerve and artery are at risk in this type of of the following would be an indication for
injury? x-ray imaging of the affected ankle?
a) Ulnar nerve and axial artery a) Inability to weight bear immediately after
b) Axial nerve and radial artery the injury
c) Median nerve and brachial artery b) Inability to weight bear immediately after
d) Radial nerve and brachial artery the injury and pain in the malleolar zone
e) Musculocutaneous nerve and axillary c) Pain in the malleolar zone and body
artery tenderness over the posterior aspect of
f) Posterior interosseous nerve and dorsal the lateral malleolus
scapular artery d) Bony tenderness over the posterior
aspect of the medial malleolus
e) Bruising over the anterior aspect of the
2) A 23 year old male is playing soccer on a medial malleolus
muddy field when he is tackled from the
side. He immediately grabs his lower right
leg and to his horror feels a bone 4) A 23 year old male is involved in a multiple
protruding through the skin. At the hospital vehicle collision during morning rush hour.
he is diagnosed with an open fracture of the After arriving to the hospital via ambulance
right tibia. Which of the following initial around noon, it is discovered that the
antibiotic regimens is appropriate: gentleman is suffering, among a variety of
a) Cefazolin for 24 hours other minor injuries, from an open fracture
b) Cefazolin for 48 hours of his left distal radius. Upon questioning, it
c) Cefazolin, gentamicin and penicillin for is revealed that this gentleman is quite
72 hours healthy and taking no medications. He
d) Cefazolin and gentamicin for 48 hours denies any allergies. On physical exam,
e) Gentamicin for 48 hours the open fracture is quite obvious, although
the laceration is <2cm. A neurovascular
exam is unremarkable for any worrisome
findings. At this point, the most important
step in management would be:
a) Immobilize in a cast and follow-up in 1
week to reassess healing
b) Proceed straight to the OR for an I and
D, followed by ORIF
c) Inquire about tetanus status and
respond accordingly
d) Administer antibiotics that target gram
negative and anaerobic bacteria
e) Inquire about tetanus, give antibiotics
and book the patient for an
intraoperative I and D, and possible
ORIF, to be done the following day
5) You are the on call orthopedic surgery 7) An 18 year-old motorcyclist presents in the
resident during a stormy winter night. emergency department following an
During the evening hours, you are paged to accident. He has a compound tibia and
see a 73 year old lady who slipped on a fibula fracture of the right leg and on
patch of black ice. She is complaining of examination the right leg has no pulses.
severe right hip pain and nothing else. Your immediate treatment should be:
Upon questioning, you find out she suffers a) Immediate angiogram
from hypertension and osteoporosis; b) Immediate surgery
moreover, you also discover that she has c) Casting and/or splinting
broken her right distal radius on 2 other d) Reduction and splinting
occasions û fixed with closed reduction on e) X-ray
both occasions. Her osteoporosis is being
managed with calcium, vitamin D and an
unknown bisphosphonate. Her anti- 8) Which of the following is the most serious
hypertensive medication is unknown, but complication of a displaced supracondylar
she admits that her blood pressure is well fracture of the humerus?
controlled. There are no other medications. a) Compartment syndrome of the forearm
The history also reveals that she has had b) Failure to heal
an appendectomy and C-section in the c) Healing in a non-anatomical position
past, has no allergies, has never smoked d) Injury to the median nerve
and her last meal was lunch time. On e) Significantly limited range of elbow
physical examination, her right leg is motion
shortened and in marked external rotation.
Her right leg, as well as her other
extremities, are neurovascularly intact. 9) All of the following statements regarding
The rest of the exam is unremarkable. You knee injuries are correct EXCEPT:
send this lovely lady for an X-ray which a) Locking of the knee may be due to a
shows a grade 4 subcapital fracture of her torn meniscus
right hip. Her intraoperative treatment b) Minor tears of the medial collateral
would include: ligament can be treated with brief
a) Hemiarthroplasty of her right hip immobilization then range of motion and
b) Total arthroplasty of her right hip strengthening exercises
c) Dynamic hip screw c) Lateral meniscus tears are more
d) Gamma nail common than medial meniscal tears
e) ORIF femoral neck d) Anterior cruciate ligament tears may
give a positive Lachman test
e) A knee dislocation may be associated
6) A 53 year old male suffers an undisplaced with major ligament damage
fracture of his medial malleolus. It is
immobilized with a plaster cast. He has a
follow up visit in the fracture clinic in 2 10) An 83 year-old man has fallen while walking
weeks time to assess bone healing. You down stairs. He is brought to the
explain to him the potential complications of emergency department with a 3-part
this fracture, including all of the following intertrochanteric hip fracture. Which of the
EXCEPT: following procedures would you choose to
a) Mal-union perform?
b) Non-union a) Hemiarthoplasty
c) Infection b) Total hip replacement
d) Fat embolism c) Multiple pin fixation
e) DVT d) Bipolar arthroplasty
e) Pin and plate
11) A 16 year-old female fell while roller-blading 15) All of the following have been associated
on her outstretched right hand. At a nearby with posterior shoulder dislocation
emergency department X-rays confirmed EXCEPT:
the diagnosis of a closed Colles fracture. a) Ethanol
The proper reduction technique for this b) Electricity
wrist fracture is which of the following: c) Exercise
a) Slight extension, full pronation, and full d) Epilepsy
ulnar deviation e) Encephalitis
b) Slight flexion, full supination, and full
radial deviation
c) Slight extension, full supination, and full 16) Management of an open fracture should
ulnar deviation always include each of the following
d) Slight flexion, full pronation, and full EXCEPT:
ulnar deviation a) Assessment of neurovascular status
e) Slight extension, full pronation, and full b) Reduction and fixation of fracture
radial deviation c) Irrigation and debridement of wound
d) Application of sterile dressing
e) Application of topical antibiotics
12) Which of the following is least likely to
cause avascular necrosis:
a) Sickle cell disease
b) Septic arthritis
c) Steroid use
d) Constrictive dressings
e) Post-traumatic fracture

13) A 24 year-old football player severely


fractures his ankle while playing in a game
and subsequently requires ORIF treatment.
Indications for ORIF treatment of an ankle
fracture include all of the following
EXCEPT:
a) A fracture-dislocation
b) Undisplaced fracture with Grade II ATFL
tear
c) Trimalleolar fracture
d) Unstable talar tilt
e) Unable to maintain a closed reduction

14) Which of the following radiographic features is


most consistent with osteoarthritis of the
knee?
a) Marginal erosions
b) Juxta-articular osteopenia
(demineralization)
c) Loss of articular cartilage with narrowing of
the radiologic joint space
d) Osteonecrosis (avascular necrosis) of
the medial femoral condyle
e) Syndesmophyte formation
ANSWERS

1. D 4. C 7. D 10. E 13. B
2. C 5. B 8. A 11. D 14. C
3. C 6. D 9. C 12. D 15. C
16. E
Otolaryngology


1) The following statements regarding noise 5) What is the typical audiogram finding in
are true EXCEPT: noise induced hearing loss:
a) Temporary threshold shift recovers a) Conductive loss in the low frequencies
following cessation of noise exposure b) Conductive loss at 2000 Hz
b) Permanent threshold shift is c) Sensorineural loss at 2000 Hz
characterized by a progressive pattern of d) Sensorineural loss at 4000 Hz
hearing loss e) Mixed broad spectrum hearing loss
c) Most cases of permanent threshold shift
are surgically treatable
d) Higher frequency noise is more 6) Which of the following is not a complication
damaging than low frequency noise of untreated otitis media?
e) None of the above a) Cholesteatoma
b) Meningitis
c) Tympanic membrane perforation
2) The frequencies most necessary for the d) Trigeminal neuralgia
understanding of speech extend from about: e) Conductive hearing loss
a) 20-20 000 Hz
b) 400-4 000 Hz
c) 250-8 000 Hz 7) Complications of tonsillectomy include all of
d) 100-5 000 Hz the following EXCEPT:
e) None of the above a) Secondary hemorrhage
b) Severe otalgia
c) Quinsy
3) Conductive hearing loss is a symptom of: d) Nasopharyngeal stenosis
a) Presbycusis e) None of the above
b) Meniere disease
c) Cholesteatoma
d) Bell palsy 8) Features characteristic of acute tonsillitis
e) Acoustic neuroma include all of the following EXCEPT:
a) Odynophagia
b) Cough
4) The following statements regarding epistaxis c) Referred otalgia
are false EXCEPT: d) Cervical lymphadenopathy
a) Epistaxis rarely occurs in children e) Dysphagia
b) It commonly results from rupture of
posterior placed nasal vessels
c) It may be treated by ligation of the 9) A 6 month-old baby has had mild inspiratory
ipsilateral internal carotid artery stridor for the last 2 months. The most likely
d) Epistaxis may be treated by cautery of cause of this is:
Little's area with silver nitrate a) Laryngomalacia
e) It is not a common cause of emergency b) Acute epiglottitis
admission to ENT wards c) Croup
d) Tonsillar hypertrophy
e) Foreign body aspiration
10) Which of the following is true regarding 14) Which of the following is an absolute
mandibular fractures? indication for a tonsillectomy?
a) They are predominantly unilateral a) Airway obstruction
b) Compound fractures are rarely intraoral b) Recurrent (>5) episodes of tonsillitis
c) They may manifest with numbness in the c) Peritonsillar abscess
V2 facial nerve distribution d) Tonsillar hypertrophy
d) Malocclussion of teeth is a common sign e) Halitosis
e) They seldom result in trismus

15) Which of the following is not a cause of


11) Nasopharyngeal carcinoma: sensorineural hearing loss:
a) Is most common in people from south- a) Ossicular discontinuity
east Africa b) Ototoxicity
b) Presents early with nasal pain c) Méniére disease
c) Is treated by wide surgical excision d) Noise
d) Is monitored by measurement of e) Presbycusis
Epstein-Barr virus antibodies
e) Is curable in 90% of patients
16) Which of the following is not true of
Meniere’s disease?
12) Factors which contribute to the development a) Characterized by quadrad of vertigo,
of squamous cell cancers of the head and hearing loss, tinnitus and aural fullness
neck include all of the following EXCEPT: b) Vertigo burns out with time
a) Cigarette smoking c) Can be treated with diuretics
b) Alcohol ingestion d) It is the result of an abnormal buildup of
c) Aging potassium
d) Exposure to particular emission from e) The vertigo lasts for seconds
diesel engines
e) Solar irradiation
17) What is the most common type of thyroid
cancer?
13) Which of the following statements about a) Medullary
aspiration of a peanut into the b) Papillary
tracheobronchial tree is FALSE? c) Follicular
a) The peanut is more likely to be in the left d) Lymphoma
lower lobe bronchus e) Anaplastic
b) Expiratory wheeze is the most likely
finding on physical exam
c) The peanut should be removed with the
patient under general anesthesia through
an open bronchoscope with forceps
designed to grasp peanuts
d) A chest x-ray may show atelectasis distal
to the blocked bronchus
e) Pneumonia may be a complication
ANSWERS

1. C 5. D 9. A 13. A 17. B
2. B 6. D 10. D 14. A
3. C 7. B 11. D 15. A
4. D 8. C 12. D 16. E
Pediatrics


1) 6-month-old boy is brought to you with white 6) Children with fifth disease (erythema
patches on mucous membranes of mouth, infectiosum, caused by parvovirus B19) do
the most likely diagnosis is: NOT have:
a) Herpes simplex a) Fever
b) Candida infection b) “Slapped-cheek” rash
c) Molluseum contagiosum c) “Lacy” rash on the trunk
d) Staphylococcal folliculitis d) Rash on palms and soles

2) A child is born with jaundice and has 7) As opposed to nutritional marasmus,


seizures, CT head shows intracranial kwashiorkor is characterized by:
calcifications, the most likely diagnosis is: a) Anorexia
a) Hepatitis B b) Alopecia
b) Congenital rubella c) Severe muscle wasting
c) Congenital syphilis d) Edema
d) Congenital toxoplasmosis

8) Anterior bowing of the tibias (saber shins) is


3) Of the following, the most common associated with:
presentation of child abuse are: a) Beriberi
a) Burns b) Multiple sclerosis
b) Bruises c) Congenital syphilis
c) Bone fractures d) Duchenne muscular dystrophy
d) Intraabdominal injuries

9) 2 year old child develops barking cough and


4) 3 year old child has respiratory distress, stridor and symptoms of an upper
stridor and cough, chest X-ray shows respiratory infection. The most likely
subglottic narrowing (steeple), the most diagnosis is:
likely cause is: a) Croup
a) Klebsiella b) Sinusitis
b) Streptococcus c) Epiglottitis
c) Parainfluenza virus d) Pertonsillar abscess
d) Staphylococcus aureus

10) The most common cause of otitis externa in


5) A premature infant has abdominal children and adults is:
distention, X-ray shows bubbly intestinal a) Proteus
wall and hepatic portal venous gas, your b) Candida
diagnosis is: c) Escherichia coli
a) Celiac disease d) Pseudomonas aeruginosa
b) Cystic fibrosis
c) Ulcerative colitis
d) Necrotizing enterocolitis
11) One year old child has septic arthritis, the 17) A baby turns blue whenever it stops crying.
most likely cause is: The most likely diagnosis is:
a) Escherichia coli a) Lung abscess
b) Neisseria meningitidis b) Cystic fibrosis
c) Staphylococcus aureus c) Mitral stenosis
d) Haemophilus influenzae d) Choanal atresia
e) Tetralogy of Fallot

12) Ambiguous genitalia in infants is most


commonly due to: 18) Which of the following should be ruled out in
a) Congenital adrenal hyperplasia a 15 year old boy with gynecomastia?
b) Testicular feminization a) Down's syndrome
c) Ovarian tumor b) Edward's syndrome
d) Pituitary adenoma c) Turner's syndrome
e) Gonadal dysgenesis d) Fragile X syndrome
e) Klinefelter's syndrome

13) Patients with Klinefelter syndrome have:


a) Small testes 19) Children with pectus excavatum (funnel
b) Short stature chest) are most likely to develop:
c) 47 XYY karyotype a) Kyphosis
d) Increased facial hair b) Sarcoidosis
e) Decreased serum estradiol c) Osteosarcoma
d) Cystic hygroma
e) Achondroplasia
14) Diagnosis of hemolytic-uremic syndrome is
most likely to be confused with:
a) Thalassemia 20) X-ray of lateral neck of a 6 year old child
b) Pyelonephritis shows thumb-print sign, the most likely
c) Hydronephrosis diagnosis is:
d) Polycystic kidney disease a) Croup
e) Thrombotic thrombocytopenic purpura b) Epiglottitis
c) Cystic fibrosis
d) Bronchial asthma
15) Of the following, the best medication for e) Epiglottal cancer
absence seizures (“petit mal”) is:
a) Phenobarbital
b) Diazepam 21) Croup (laryngotracheobronchitis) is:
c) Ethosuximide a) Most commonly caused by fungus
d) Clonazepam b) Associated with inspiratory stridor
e) Levodopa c) Usually occur in ages 10 to 15 years
d) More common in males than in females
e) Characterized by high fever for 3 days
16) Hirschsprung's disease (congenital
aganglionic megacolon /rate of 1 in 5000
live births) is best diagnosed by: 22) A couple had a child with bilateral cleft lip
a) Sigmoidoscopy and palate, the chance of having another
b) Rectal examination child with the same defect is:
c) Rectal biopsy a) 3%
d) Barium enema b) 13%
e) Plain abdominal X ray c) 30%
d) 53%
23) Infants born to diabetic mothers often have: 29) The most common cause of acute
a) Hypercalcemia gastoenteritis in developed countries is:
b) Hyperglycemia a) Viral
c) Cardiomegaly b) Fungal
d) None of the above c) Bacterial
d) Parasitic
24) Sudden, symmetric adduction and flexion of
limbs with concomitant flexion of head and
trunk in a one year old child is most likely 30) The most common extracutaneous site of
due to: involvement of chickenpox in children is:
a) Petit mal a) Genitourinary system
b) Grand mal b) Cardiovascular system
c) Febrile convulsions c) Central nervous system
d) Infantile spasms (West's syndrome) d) Gastrointestinal system

25) The most common malignant hepatic


tumour in a one year old child is:
a) Hepatoblastoma
b) Cholangiocarcinoma
c) Fibrolamellar carcinoma
d) Hepatocellular carcinoma

26) What would you order for a 12 year old girl


with recurrent episodes of edema lasting 48-
72 hours affecting the face?
a) Monospot test
b) Lymph node biopsy
c) Muscle enzyme levels
d) C1 inhibitor levels

27) Ewing’s tumor is a:


a) Bone cyst
b) Benign tumor
c) Malignant sarcoma
d) A form of multiple myeloma

28) Infants with congenital heart diseases may


present with:
a) Hemoptysis
b) Hepatomegaly
c) Both
d) Neither
ANSWERS

1. B 7. D 13. A 19. A 25. A


2. D 8. C 14. E 20. B 26. D
3. B 9. A 15. C 21. B 27. C
4. C 10. D 16. C 22. A 28. B
5. D 11. C 17. D 23. C 29. A
6. D 12. A 18. E 24. D 30. C
Plastic Surgery


1) A 30 year old man was trapped in a burning 3) A 50 year old healthy man who recently
house. When he was rescued and brought underwent day surgery for right inguinal
to hospital, it was found that he had first hernia repair presents with fever and pain
degree burns to his torso, second degree over his incision area. On examination, it
burns to the anterior aspect of both his arms was found that there is a poorly defined
and third degree burns to both his legs. His erythematous region around his incision that
weight is approximately 70 kg. You are the is tender to palpation with no areas of
emergency physician that first received him necrosis. Which of the following organism is
in the Emergency Room, which fluid most likely the cause of his infection?
resuscitation would you initiate if he a) Escherichia coli
presented 1 hour after his burn injury. b) Pseudomonas aeruginosa
a) Ringer's Lactate 525 ml/hr c) Streptococcus pyogenes
b) Ringer's Lactate 788 ml/hr d) Clostridium perfringens
c) Ringer's Lactate 900 ml/hr e) Pasteurella multocida
d) Ringer's Lactate 1420 ml/hr
e) Ringer's Lactate 1620 ml/hr

2) A 56 year old woman who works as a 4) You are working in the emergency
medical secretary comes into the office department when a burn victim is brought in.
complaining of pain in her left hand. After a Which of the following is not an indication for
detailed history and physical, it was transfer to a burn center:
concluded that her pain is most likely a) A 40 year old man who fell asleep while
attributed to carpal tunnel syndrome. Which smoking and sustained a mixed 2º and
of the following is not a feature of carpal 3º burn to 15% of his body including his
tunnel syndrome? right hand, arm and chest
a) Numbness and pain in the affect hand b) A five year old boy with a mixed 2º and
leading to frequent night awakenings 3º burn to 15% total body surface area
b) Decreased ability to abduct the thumb on his back from a scald injury
c) Decreased sensation in the 4th and 5th c) A 30 year old woman who was rescued
digits from a burning building. She has a 5%
d) Flattening of the thenar eminence TBSA mixed 2º and 3º burn on her right
e) Percussion over the median nerve at the forearm and 12% mixed 2º and 3º on her
wrist reproduces symptoms thigh. She was also trapped in a smoke-
filled room and appears to be in
respiratory distress
d) A 45 year old factory worker who
sustained a 0.5% total body surface area
partial thickness burn to his left forearm
after coming into contact with
hydrofluoric acid
e) A 58 year old woman who sustained a
high voltage electrical injury
5) A 32 year old man comes into the 8) In which of the following are systemic
emergency department after a sustaining antibiotics not indicated:
facial blow injuries from a fight. The CT a) Animal bite to hand
head reveals that he has sustained facial b) Laceration over metacarpalphalangeal
bone fractures. Which of the following is an joint sustained after punching someone
indication for him to have immediate in a fight
surgery: c) Cellulitis
a) He has a displaced nasal bone fracture d) Burn wound to 75% of the dorsal aspect
b) His CT head reveals an orbital floor of the hand
fracture with no soft tissue entrapment e) Acute suppurative tenosynovitis
c) He has ptosis and proptosis of his left
eye and is complaining that he cannot
see out of this eye even when the eyelid 9) Which of the following is not a finding in
is held open mandibular fractures?
d) He has an unstable fracture of the a) Palpable step along mandible
zygomatic complex b) Loose teeth
e) He has a bilateral LeFort I fracture c) Trismus
d) Numbness in V2 distribution
e) Malocclusion
6) Which of the following is not a sign or
symptom of carpal tunnel syndrome: 10) A 43 year-old woman is brought to the
a) Loss of sensation to the proximal palm Emergency Department after being burned
b) Positive Phalen test in a house fire. You estimate first degree
c) Positive Tinel sign burns to 20% of her body, second degree
d) Pain involving the thumb, index, long, burns to 11% of her body, and third degree
and part of the ring digits burns to 9% of her body. She weighs 60 kg
e) Delayed transit time on nerve conduction and is 120 cm tall. What IV therapy would
study you begin immediately?
a) Normal saline at 200cc/hr for 24 hours
b) Normal saline at 400cc/hr for 24 hours
7) A 35 year-old intoxicated man presents to c) Normal saline at 150cc/hr for 8 hours,
the Emergency Department with a deep then 75cc/hr for the next 16 hours
laceration to his right distal forearm after d) Normal saline at 300cc/hr for 8 hours,
putting his hand through a window. On then 150cc/hr for the next 16 hours
examination, he has no sensation to his little e) Normal saline at 600cc/hr for 8 hours,
finger and the ulnar aspect of his ring finger. then 300cc/hr for the next 16 hours
Power on abduction and adduction of all
fingers is markedly decreased. He is unable
to flex the distal joint of his little finger. When
he flexes his wrist, his hand deviates
radially. The examination was otherwise
normal. What structure(s) have been
damaged?
a) Ulnar nerve
b) Flexor carpi ulnaris and flexor digitorum
profundus muscles
c) Median nerve
d) Radial nerve
e) Brachioradialis
11) Which of the following statements is 13) A 45 year old woman with a history of
incorrect with respect to wound healing? hypothyroidism comes to your office
a) Epithelialization can occur within 24 complaining of problems with extending the
hours following primary closure of a middle finger of her left hand for the last two
wound months. She complains of a snapping of
b) Maximum wound strength is often her finger when she forces extension and on
achieved after 2 years examination, you can feel a nodule on the
c) Wounds continue to gain strength after palmar aspect of her left hand proximal to
collagen synthesis has reached an the third MCP joint. Based on your history
equilibrium and physical exam, you conclude that her
d) Wound contraction is mediated by symptoms are most likely attributed to
myofibroblasts stenosing tenosynovitis. Which one of the
e) The incidence of wound infection following options is incorrect:
increases with healing by secondary a) You suspect that her symptoms are
intention arising at the A-1 pulley. This is the
most common site for this condition to
arise
b) The ring finger is one of the most
12) Which of the following is not a stage in the commonly affected fingers
development of pressure ulcers? c) This condition is also associated with
a) Ischemia rheumatoid arthritis and diabetes mellitus
b) Congestion d) The most appropriate therapeutic option
c) Hyperemia at this time would be conservative
d) Ulcer treatment with NSAIDs and steroid
e) Necrosis injections
e) The most appropriate therapeutic option
at this time is surgical incision of the A-1
flexor tendon sheath
ANSWERS

1. C 4. D 7. A 10. D
2. C 5. C 8. D 11. E
3. C 6. A 9. D 12. B
13. E
Community and Population Health

1) The following is an example of primary 4) Which of the following statements regarding


prevention: measurements of the effectiveness of
a) Routine mammography to detect early interventions is false?
breast cancer tumour growth a) Relative risk reduction (RRR) is defined
b) Routine Pap smear to detect early as the proportional reduction in rates of
endocervical cytological changes bad outcomes between experimental
c) ACE inhibitor to treat hypertension and control participants in a trial
d) Vaccination to prevent diphtheria b) Absolute risk reduction (ARR) is defined
e) Lithium to manage mood symptoms as the absolute arithmetic difference in
associated with bipolar disorder rates of bad outcomes between
experimental and control participants in
a trial
2) The number one cause of potential years of c) The number needed to treat (NNT) is
life lost (PYLL) in Canada is: defined as the inverse of the absolute
a) Neoplasm risk reduction (ARR) -- i.e. 1/ARR
b) Circulatory disease d) Compliance is defined as the degree to
c) Unintentional injuries which a patient adheres to a treatment
d) Suicide plan
e) Respiratory disease e) Effectiveness is defined as the extent to
which a specific intervention produces a
beneficial result under ideal conditions
3) Which of the following statements regarding achieved in a clinical trial
sensitivity of Test X for Disease Y is true?
a) It is defined as the true positives divided
by the false negatives 5) Which is not a result of chronic lead
b) It refers to the proportion of the people exposure?
who do not have Disease Y whose Test a) Peripheral motor neuropathy
X is negative b) Nephrotoxicity
c) If Test X is sensitive and is found to be c) Hemolytic anemia
negative, it can be used to rule out a d) Microcytic anemia
disease hypothesis
d) Sensitivity and specificity are not useful
in determining the likelihood ratio of a 6) Which is not a function of municipal public
test health units?
e) The positive predictive value of Test X is a) Tobacco legislation enforcement
the inverse of the sensitivity of Test X b) Inspections of restaurants
c) Approval of new medications
d) Reportable disease surveillance and
follow-up

7) Which of the following is not a reportable


disease?
a) HIV
b) Measles
c) Tetanus
d) Streptococcal pharyngitis
8) Which is an example of a propagated 12) Lead exposure typically results in:
epidemic? a) Chronic dermatitis
a) Exposure is brief and essentially b) Resting and intention tremor
simultaneous after exposure to a c) Extensor muscle weakness
common source d) Arrhythmias
b) Begins with only a few exposed e) Cerebellar ataxia
persons, but is maintained by person-
to-person transmission
c) Exposure to a common source lasts for 13) Which of the following statements
a period of days to weeks concerning the Worker’s Compensation Act
is true?
a) The worker reserves the right to sue the
9) Which is an example of a cohort study? employer for negligence
a) Subjects are sampled, and as a group b) Funding is provided by the provincial
are classified on the basis of presence government
or absence of exposure to a particular c) The worker is guaranteed payment from
risk factor the first day of injury/illness if it is
b) Samples a group of people who already deemed to be work-related
have a particular outcome and d) The Worker’s Compensation Board is
compares them to a similar sample an independent, private agency
group without that outcome e) None of the above
c) Status of individual with respect to
presence and absence of both
exposure and disease assessed at one 14) Which of following statements regarding
point in time radiation is false?
d) Random distribution of baseline a) Natural background radiation accounts
characteristics and treatment between for about half of a typical person’s
groups exposure
b) Ionizing radiation causes intestinal villi
to become denuded
10) Which is not true of the WSIB? c) Exposure to non-ionizing radiation may
a) The WSIB decides benefits for workers result in cataracts
b) The employers pay for WSIB costs d) Ionizing radiation results in an increased
c) Once a claim has been made, incidence of neoplasia such as lung and
physicians are required to provide the thyroid
WSIB with information about a worker's e) None of the above
health
d) Workers are able to sue the employer
up to a maximum of $100,000 in cases 15) Which statement concerning vibration
of negligence induced white finger disease is false?
a) Early symptoms include tingling and
numbness of the fingers when at rest
11) Which one of the following is not a typical b) Swelling of the fingers over knuckles
feature of asbestosis? may be an early feature
a) Increased risk of cancer c) Cold, damp conditions may precipitate
b) Pleural thickening and calcification symptoms
c) Interstitial fibrosis d) The affected area eventually spreads to
d) Obstructive pattern on pulmonary involve all fingers
function tests e) None of the above
e) None of the above
16) In a cohort study of disease “X” in people 19) The following indicate the results of
with risk factor “Y” versus those who are screening test “Q” in screening for disease
without risk factor “Y”, the following results “Z”:
were obtained: Disease Z
+ -
X no X Total + 40 10 50
Y 80 20 100 Screen - 30 120 150 Q
no Y 50 50 100 a)70 130 200

The relative risk of developing “X” in “Y” The specificity of test “Q” would be:
versus no “Y” is: a) 40/70
a) 80 x 50 = 4 c) 50 x 80 = 4 b) 120/130
50 x 20 20 x 50 c) 40/50
d) 120/150
b) 50 _ 20 = 0.3 d) 80/100 = 1.6 e) 40/130
100 100 50/100

e) it is not possible to calculate risk in a 20) The positive predictive value would be:
cohort study a) 40/70
b) 120/130
17) The attributable risk of factor “Y” to disease c) 40/50
“X” would be: d) 120/150
a) 80/100 = 1.6 c) 80 x 50 = 4 e) 70/200
50/100 50 x 20

b) 80 _ 50 = 0.3 d) 50 x 20 = 0.25 21) To determine an odds ratio one would have


100 100 80 x 50 to perform which of the following studies?
a) A cross sectional/prevalence study
e) it is not possible to calculate b) A randomized controlled trial
atrributable risk in a cohort study c) A cohort study
d) A case study
e) A case control study
18) A group of 50 people are exposed to virus
“A”. Of those 50 people, 9 develop a mild
infection, 10 become seriously ill, and 3 die. 22) Examples of secondary prevention would
The attack rate of virus “A” in the population include all of the following EXCEPT:
would be: a) Pap smear for cervical cancer
a) 22/50 b) Chemoprophylaxis in a recent TB
b) 9/50 converter
c) 10/50 c) Proctoscopy for rectal cancer
d) 19/50 d) Immunization for Haemophilus
e) 13/50 influenzae B
e) Mammography for breast cancer
23) Alpha error is: 27) The component of Canada's health care
a) The probability of declaring a difference system that receives the highest
to be absent when it in fact is present percentage of the health care budget is:
b) The probability of declaring a difference a) Hospitals
to be present when it is not b) Physician fees
c) The probability of declaring a difference c) Drug benefit plans
to be absent when it is indeed absent d) Laboratory services
d) The probability of declaring a difference e) Administration
to be present when it does exist

28) The British North America Act (BNA):


24) Which one of the following descriptors of a a) Was mainly concerned with health care
diagnostic test is influenced by the legislation
prevalence of the disease being tested for: b) Granted exclusive powers over health
a) Specificity care to the provinces, including powers
b) Sensitivity over marine hospitals
c) Accuracy c) Replaced the Canada Health Act
d) Positive predictive value d) Applied to British Columbia only
e) Reliability e) None of the above

25) Which of the following statements regarding 29) Regarding health expenditure and health
the measurement of health and disease in a outcomes:
population is true? a) The U.S. has the greatest health
a) A rate is the number of times an event expenditure and the lowest infant
has occurred during a certain time mortality rates
interval divided by the number of b) There is a positive association between
persons at risk during the same interval national expenditure on health and GDP
b) When calculating a ratio, the numerator c) Increased national health expenditure
is a portion of the denominator always increases health status of a
c) A ratio is the number of times an event country
has occurred during a certain time d) All of the above
interval multiplied by the number of e) None of the above
times an event has occurred during the
same interval
d) When calculating a rate, the numerator 30) Which of the following is the most important
is not a portion of the denominator justification for population screening
e) None of the above programs for a specific disease?
a) Early detection of the disease of interest
is achieved
26) In 1990, which country spent the least on b) The specificity of the screening test is
health care as a percentage of GDP (gross high
domestic product): c) The natural history of the disease is
a) Canada favourably altered by early detection
b) France d) Effective treatment is available
c) Sweden e) The screening technology is available
d) UK
e) USA
31) Regarding the regulation of health 34) All of the following statements are true
professionals, provincial colleges of EXCEPT:
physicians and surgeons: a) The data collected on a death certificate
a) Have the advancement of the public is uniform and in conformity with WHO
interest as their primary goal guidelines
b) Protect the public from incompetent or b) Section 7 of the Coroner’s Act states
unfit MDs that the coroner’s office must be notified
c) Act as licensing bodies for MDs if a patient dies after some mishap such
d) Do not advance the professional and as leaving an instrument in the body at
political interests of MDs surgery
e) All of the above c) Diseases which must be reported to the
local medical officer of health include
AIDS, food poisoning, influenza and
32) Active immunization was important in gonorrhea
control of each of the following childhood d) A tuberculin reaction greater than 5 mm
communicable diseases EXCEPT: is considered positive in all individuals
a) Diphtheria e) All of the above
b) Polio
c) Measles
d) Scarlet fever 35) In describing the leading causes of death in
e) Pertussis Canada, two very different lists emerge,
depending on whether proportional
mortality rates or person-years of life lost
33) All of the following statements are true (PYLL) are used. This is because:
EXCEPT: a) One measure uses a calendar year and
a) One indirect measure of a population’s the other a fiscal year to calculate
health status is the percentage of low annual experience
birth weight neonates b) One measure includes morbidity as well
b) Accidents are the largest cause of as mortality experience
potential years of life lost in Canada c) Both rates exclude deaths occurring
c) The Canadian population is steadily over the age of 70
undergoing rectangularization of d) Different definitions of “cause of death”
mortality are used
d) Morbidity is defined as all health e) One measure gives greater weight to
outcomes excluding death deaths occurring in younger age groups
e) The neonatal mortality rate is the
number of infant deaths divided by the
number of live births multiplied by 1000 36) Differentiation between a point-source
epidemic and a progressive (propagated)
epidemic is made by:
a) Considering the characteristics of the
infectious agent
b) Determining the level of immunity in the
community
c) Determining the number of persons
infected and calculating the attack rate
d) Plotting the distribution of cases by time
onset
e) None of the above
37) The occurrence of an illness at a rate of 41) Which of the following types of studies
above that expected is called: usually provides only a measure of
a) Hyperendemic prevalence?
b) Epidemic a) Descriptive
c) Endemic b) Cross-sectional
d) Enzootic c) Randomized controlled trial
e) Pandemic d) Cohort
e) None of the above

38) Each of the following statements applies to


case control studies EXCEPT: 42) The major advantage of cohort studies over
a) Starts with disease case-control studies is that:
b) Suitable for rare diseases a) They take less time and are less costly
c) Relatively inexpensive b) They can utilize a more representative
d) Prolonged follow-up required population
e) There may be a problem in selecting c) It is easier to obtain controls who are not
and matching controls exposed to the factor
d) They permit estimation of risk of disease
in those exposed to the factor
39) A clinician who has been examining the e) They can be done on a “double-blind”
patterns of mortality in your community basis
says that the rates for heart disease and
lung cancer are higher in this community
than in an adjacent community. Which of 43) All of the following statements concerning
the following questions should you ask occupational health are true EXCEPT:
first? a) Disorders of reproduction are among the
a) How did the clinician choose the top 10 work-related diseases and
comparison community? injuries
b) Have the rates been standardized for b) Most workers are covered by both
age? federal and provincial legislation with
c) Are tobacco sales significantly different respect to workplace health and safety
in the two communities? c) Skin problems and hearing problems
d) Are the facilities to treat these diseases together are responsible for half of WCB
comparable in the two areas? claims
e) Are the numbers of deaths comparable d) A complete occupational medical history
in each area? includes investigation of the temporal
relationship between symptoms and
exposure
40) The purpose of randomization is to:
a) Make sure that there are equal numbers
of men and women in test and control
groups
b) Increase the chances of getting a
statistically significant difference
c) Ensure that the numbers of cases and
controls are equal
d) Limit bias
e) All of the above
44) Which of the following statements 47) The effectiveness of a preventative
concerning exposure to solvents in the measure is assessed in terms of:
workplace is true? a) The effect in people to whom the
a) Each solvent compound has a specific measure is offered
antidote that can be used to treat b) The effect in people who comply with
exposure the measure
b) A prominent symptom of solvent c) Availability with the optimal use of
exposure is memory loss resources
c) Some solvents can cause skin dryness d) The cost in dollars versus the benefits in
and loss of subcutaneous adipose improved health status
tissue e) All of the above
d) Solvents do not affect the bone marrow
e) All of the above
48) All of the following statements about the
Canada Health Act (1984) are true
45) In 1981, the crude birth rate in Ontario was EXCEPT:
approximately 14 per 1000 and the crude a) It did not define all medically necessary
death rate was 7 per 1000. The estimated hospital and physician services
rate of net migration was –1 per 1000. The b) The CHA replaced the Hospital
growth rate of the province, per 1000 Insurance and Diagnostic Services Act
population was: of 1957
a) 6 c) The CHA banned all forms of extra
b) 7 billing
c) 8 d) According to the CHA, provinces must
d) 20 meet all the terms and conditions of
e) 22 Medicare to qualify for federal transfer
payments
e) None of the above
46) All of the following statements about
environmental health are true EXCEPT:
a) Levels of toxic agents measured in the 49) Each of the following is an example of
environment may not reflect internal primary prevention EXCEPT:
organ levels a) Genetic counselling of parents with one
b) The federal government monitors the retarded child
quality and types of industrial emissions b) Nutritional supplements in pregnancy
and toxic waste disposal c) Immunization against tetanus
c) Sick building syndrome is associated d) Chemoprophylaxis in a recent tuberculin
with Pontiac fever and Legionnaire’s converter
disease e) Speed limits on highways
d) All humans have detectable levels of
PCBs
e) None of the above is true 50) The classical “epidemiological triad” of
disease causation consists of factors which
fall into which of the following categories:
a) Host, reservoir, environment
b) Host, vector, environment
c) Host, agent, environment
d) Reservoir, agent, vector
e) Host, age, environment
51) Of the five items listed below, the one which 54) During a clinical trial, the difference in the
provides the strongest evidence for success rates of two drugs was not
causality in an observed association statistically significant. This means that:
between exposure and disease is: a) There is no difference in drug
a) A large attributable risk effectiveness
b) A large relative risk b) There is a sizable probability that the
c) A small p-value demonstrated difference in the drugs’
d) A positive result from a cohort study effectiveness could occur due to chance
e) A case report alone
c) The demonstrated difference in the
drugs’ effectiveness is too small to be
52) The difference between a common point clinically meaningful
source outbreak and a propagated outbreak d) The two samples of patients on which
of illness is that: the drugs were tested came from the
a) All cases in a common point source same population
outbreak occur within one incubation e) None of the above are true
period of the exposure
b) The attack rates in propagated
outbreaks are higher 55) The incidence of a particular disease is
c) Person-to-person transmission is a greater in men than in women, but the
feature of common source outbreaks prevalence shows no sex difference. The
d) Case fatality rates in common source most probable explanation is that:
outbreaks are generally higher a) The mortality rate is greater in women
e) The source of infection in propagated b) The case fatality rate is higher in women
outbreaks is more easily contained than c) The duration of the disease is longer in
in common source outbreaks women
d) Women receive less adequate medical
care for the disease
53) Which of the following are strategies for e) This diagnosis is more often missed in
control of disease: women
a) Population immunization
b) Contact tracing to offer treatment to all
who could be infected 56) All of the following statements about
c) Monitoring increases in the population of statistical tests are true EXCEPT:
certain disease vectors a) Linear regression is used to describe
d) Having physicians report curable, the relationship between two continuous
potentially serious diseases variables
e) All of the above are true b) A confidence interval is a range of
values giving information about the
precision of a measurement
c) ANOVA tables are used to make
comparisons among the means of 3 or
more groups simultaneously
d) In a normal distribution, the mean,
median and mode are equal
e) The chi-square test evaluates the
statistical significance of 2 or more
percentages of categorical outcomes
57) All of the following are responsibilities of 61) Which of the following is not in the top 10
local public health units in Canada leading causes of death in Canada?
EXCEPT: a) Respiratory diseases
a) Communicable disease control b) Malignant neoplasms
b) Health education c) Diseases of the heart
c) Investigation of sudden death d) Accidents
d) Immunization e) Viral hepatitis
e) Health promotion

62) Which of the following will increase the


58) Who is ultimately responsible for the quality power of a study?
of care in a hospital? a) Increase beta
a) Chief executive officer b) Analyze data using more powerful
b) Board of trustees statistical tests
c) Medical director c) Change from z-test to a t-test
d) Chief of staff d) Increase sample size
e) Attending physicians e) None of the above

59) Which of the following is not one of the 5


Terms and Conditions of Medicare?
a) Portability
b) Flexibility
c) Universality
d) Comprehensive coverage
e) Accessibility

60) You are given this data pertaining to an


outbreak of diarrhea in a daycare:

Age Number of Number of


Children Diarrhea
1 25 20
2 20 10
3 40 10
4 40 5
5 20 4
6+ 25 1

What is the attack rate of this illness?


a) 25% d) 40%
b) 30% e) 66%
c) 33%
ANSWERS

1. D 13. C 25. A 37. B 49. D


2. A 14. E 26. D 38. D 50. C
3. C 15. E 27. A 39. B 51. B
4. E 16. D 28. E 40. D 52. A
5. C 17. B 29. B 41. B 53. E
6. C 18. A 30. C 42. D 54. B
7. D 19. B 31. E 43. B 55. C
8. B 20. C 32. D 44. C 56. E
9. A 21. E 33. E 45. A 57. C
10. D 22. D 34. D 46. B 58. D
11. D 23. B 35. E 47. A 59. B
12. C 24. D 36. D 48. C 60. C
61. E
62. D
Psychiatry

1) A 40 year old males states he is feeling 3) Which benzodiazepine has the shortest half-
down and is in conflict with his wife. You life in the body?
elicit he has always been somewhat rigid; a) Triazolam
he is a conscientious employee. You note b) Clorazepate
that he is dressed very neatly and that while c) Nitrazepam
giving you his history he rambles, giving you d) Chlordizepoxide
all the details.The most likely personality e) Diazepam
type described here is:
a) Narcissistic 4) All of the following statements are true about
b) Dependent depression in the elderly EXCEPT:
c) Passive-aggressive a) Somatic symptoms are frequently
d) Self-defeating (masochistic) substituted for emotional symptoms
e) Obsessive-compulsive b) Tricyclics should not be used because of
cardiotoxicity
2) In panic disorder, studies suggest the most c) Precipitating events are usually related
effective and long-lasting treatment is: to loss
a) Behaviour therapy d) Some cases may benefit from ECT
b) Psychoanalysis e) May present with abnormalities in
c) Diazepam cognitive function
d) Imipramine
e) Hypnotherapy
ANSWERS

1. E 2. A 3. A 4. B
Respirology

1) A 16 year old presents to the ER with a 5) Which is more often associated with hospital
severe asthma exacerbation. All of the acquired pneumonia than community-
following may be considered as part of the acquired pneumonia?
management plan EXCEPT: a) Streptococcus pneumoniae
a) Pulmonary Function tests b) Hemophilus influenza
b) Peak Flow meters c) Pseudomonas
c) Chest X-Ray d) Chlamydia pneumoniae
d) Oral prednisone e) Mycoplasma pneumoniae
e) Counselling for better treatment
compliance
6) A 30 year-old patient with asthma complains
of daily wheezing and occasional waking at
2) A 56 year old gentleman is referred to your night with cough and chest tightness for
clinic for evaluation of an incidental lung three weeks. His usual medication is
nodule on CXR. Which of the following salbutamol two puffs tid-qid. The next step
findings makes it more likely to be benign: in management is:
a) Greater than 3cm in diameter a) Add long-term theophylline
b) Ill-defined margins b) Increase salbutamol to two puffs q4h
c) Calcification c) Add ipratropium bromide two puffs qid
d) Doubles in size on repeat CXR in 15 d) Add beclomethasone two puffs qid
months e) Discontinue salbutamol and begin
e) Clubbing prednisone 50 mg od and taper over 2
weeks

3) A 36 year old woman presents to your clinic


with suspected pulmonary hypertension. 7) Which is not a feature of asbestosis?
Which of the following conditions is more a) Increased risk of cancer
suspicious of a secondary cause for b) Pleural thickening and effusion
pulmonary hypertension? c) Interstitial fibrosis
a) Raynauds phenomenon d) Obstructive pattern on pulmonary
b) Previous use of appetite suppressant function tests
drugs e) All of the above are features of
c) An isolated history of pneumonia 5 years asbestosis
ago
d) Recurrent pulmonary emboli
e) No family history of pulmonary disease 8) A 55 year-old man with a history compatible
with chronic bronchitis presents to your
office with shortness of breath on exam. In
4) Which of the following pulmonary function the history, all of the following would be
tests most reliably discriminates “pure” anticipated EXCEPT:
chronic bronchitis from emphysema? a) A 20-year history of smoking
a) Total lung capacity b) Worsening of symptoms with exposure
b) Functional residual capacity to smog
c) Residual volume c) Worsening of symptoms with acute
d) Single breath diffusing capacity respiratory infections
e) Low at 50% vital capacity d) Recurrent episodes of pleurisy
e) Increased incidence of chronic
respiratory disease in family members
9) A 55 year-old woman with asthma is on 11) Each of the following is a correct statement
systemic steroids for one year. She about COPD EXCEPT:
develops a recent right-sided pleural a) The type of emphysema associated with
effusion. She feels unwell and tires easily. smoking is usually centriacinar
Aspiration reveals a turbid fluid, a high b) Clubbing is not a clinical feature
lymphocyte count, high LDH, low glucose, c) Long-term oral steroids should be
and a pH of 7.4. The most compatible avoided
diagnosis is: d) Smoking cessation does not lead to
a) Pulmonary embolism improvement of pulmonary function
b) Empyema e) The aim of supplemental O2 therapy is to
c) Tuberculosis provide relief of shortness of breath
d) Subphrenic abscess
e) Pancreatitis

10) Diagnosis of acute symptomatic pulmonary


embolism can be excluded when which of
the following is normal?
a) Chest x-ray
b) Ventilation-perfusion lung scan
c) Bilateral leg Doppler ultrasounds
d) Arterial blood gases
e) CT scan of the chest
ANSWERS
4. D 7. D 10. B
1. A 5. C 8. D 11. E
2. C 6. D 9. C
3. D
Rheumatology

1) Ankylosing spondylitis is associated with each 3) A 70 year-old woman visits you with a 2 day
of the following EXCEPT: history of a swollen right wrist. You aspirate
a) Sacroiliitis the wrist and find intracellular rhomboid-
b) An increased frequency of HLA B27 shaped weakly positive birefringent crystals.
c) Iritis All of the following are consistent with the
d) Symmetrical peripheral erosive arthritis diagnosis of pseudogout, EXCEPT:
e) Inflammatory bowel disease a) Chondrocalcinosis
b) Erosions seen on x-ray of the wrist joint
c) Rapid response to nonsteroidal anti-
2) Which of the following is LEAST likely to be a inflammatory drugs
cutaneous manifestation of systemic lupus d) History of a similar episode in her knee 2
erythematosus? years previously
a) Butterfly rash e) Rapid response to intraarticular
b) Discoid rash corticosteriods
c) Palpable purpura due to leukocytoclastic
vasculitis
d) Petechiae due to thrombocytopenia 4) A 22 year-old man presents with a subacute
e) Urticaria onset over 3 weeks of an asymmetrical
oligoarthritis involving the lower limbs. The
MOST LIKELY type of arthritis the patient has
is:
a) Osteoarthritis
b) Gout
c) Rheumatoid Arthritis
d) Reactive Arthritis
e) Septic Arthritis
ANSWERS

1. D 2. E 3. B 4. D
Urology


1) Patients with an organic cause for 5) A 75 year-old man with a history of nocturia
impotence are often characterized by all of has not micturated for the last 10 hours, and
the following EXCEPT: is complaining of severe lower abdominal
a) Diabetes pain. The most likely cause is:
b) Older age a) BPH
c) Intermittent difficulty b) Prostate cancer
d) Nocturnal penile tumescence absent c) Renal failure
e) None of the above d) UTI
e) Prostatitis

2) In the investigation of infertility, a normal


semen analysis MUST contain: 6) A 28 year-old male presents with painless
a) A volume of 0.5-1 mL swelling in the testicle. On examination, it
b) Morphology > 85% normal forms appears to be a rubbery, hard mass. He has
c) WBC < 1 per high power field no palpable abdominal masses. A testicular
d) Motility > 75% ultrasound confirms the diagnosis of
e) Sperm count > 20 million sperm/mL testicular tumour, and an abdominal CT
reveals normal retroperitoneal nodes. Which
statement is correct with respect to this
3) With respect to control of micturition, all of case?
the following are true EXCEPT: a) His overall prognosis is very poor
a) Damage to the cerebral cortex results in b) The testicular mass is likely a secondary
hyperactivity of the detrusor tumour
b) The basal ganglia inhibits bladder over c) A needle aspiration of the tumour is
activity indicated
c) The cerebellum coordinates emptying of d) A transcrotal approach for biopsy is
the bladder contraindicated
d) Damage to the cerebellum results in e) He will likely require chemotherapy
hypotonicity of the detrusor
e) Injury above the brainstem results in
hyperactive bladder only 7) A 34 year-old woman presents with a 3 day
history of increasing fever with chills,
sweats, nausea, and 2 episodes of vomiting.
4) Which of the following drugs will not On examination, she has costovertebral
promote urine retention? angle tenderness and urine reveals gross
a) Sympathomimetics pyuria. Initial management should include:
b) Anticholinergics a) Abdominal ultrasound
c) Cholinergic agonists b) Abdominal CT
d) TCAs c) Empirical treatment with antibiotics;
e) Smooth muscle depressants image only if she fails to respond to
therapy
d) Treat with antibiotics and perform IVP 4-
6 weeks after resolution of illness
e) Immediate DTPA renal scintigraphy
8) A 45 year-old man with a history of 11) A 40 year-old women presents to the ER
polycystic kidney disease presents with with fever, no nausea or vomiting, and left
painless gross hematuria. You order: flank pain radiating to the groin. Body CT
a) No investigation is required since the reveals 8 mm stone in the left proximal
hematuria is most likely due to the ureter. You should:
rupture of renal cyst(s) a) Send her home because renal stone is
b) No investigation at this time. Investigate benign disease
if gross hematuria persists b) Send her home with oral analgesics and
c) U/S ask her to come back if the pain does not
d) U/S, urine C&S, cystoscopy resolve in the next 24 hours
c) Admit, close monitoring, hydration and
analgesics
9) With respect to painful scrotal swelling, all of d) Admit, hydration, analgesics and IV
the following are true EXCEPT: antibiotics. (e.g. Amp and Gent), and
a) Torsion of testicular appendages will consider stenting if the symptom persists
usually subside without surgical
intervention
b) Nausea/vomiting is very common in 12) Which of the following is not a type of non-
epididymitis seminomatous germ cell tumor of the
c) Ultrasound is helpful in determining the testicle?
cause of hematocele a) Teratoma
d) Torsion most commonly occurs in young b) Embryonal carcinoma
males c) Yolk sac carcinoma
e) Orchitis may result in testicular atrophy d) Leydig cell tumour
e) Choriocarcinoma

10) Which is following statement about PSA is


true? 13) A 46-year-old male patient comes to the ER
a) Every man past the age of 50 should because of a painless swelling of his left
have an annual PSA test as a primary testis; he gives history of mild trauma a few
screen for prostate cancer days earlier. The swelling is cystic, involving
b) PSA is best used to follow disease the superior region of the testis and
progression or recurrence post- transilluminates. The diagnosis is:
operatively a) Varicocele
c) PSA is a useless test b) Tumour
d) Increased complex PSA to total PSA c) Spermatocele
ratio favors BPH over prostate cancer d) Hydrocele
e) All of the above e) Hematocele
ANSWERS

1. C 4. C 7. C 10. B 13. C
2. E 5. A 8. D 11. D
3. D 6. D 9. B 12. D

Das könnte Ihnen auch gefallen